Sunteți pe pagina 1din 91

General Science and Technology- Subject wise compilation of 60 Day Plan- 2016

Q.1) Consider the following statements regarding Bio-fertilizers


1. The microorganisms in bio-fertilizers restore the soil's natural nutrient cycle and build
soil organic matter.
2. Bio-fertilizers directly increase soil fertility by adding nutrients
3. Rhizobium is a free living nitrogen fixing bacteria
Select the correct code
a) 1 and 2
b) 2 and 3
c) Only 1
d) 1, 2 and 3
Q.1) Solution (c)
Picked from class 9th NCERT- Mention of bio-fertilizers is there in chapter 15
IMPROVEMENT IN FOOD RESOURCES
UPSC has asked questions based on such terms which are always in news. Try to know about
types of Bio-fertilizers also.
First statement is correct but second and third is not. How and Why?
First of all, bio-fertilizers are not fertilizers, which directly increase soil fertility. Rather Biofertilizers add nutrients through the natural processes of nitrogen fixation, solubilizing
phosphorus, and stimulating plant growth through the synthesis of growth-promoting
substances.
There are various groups in which Bio-fertilizers are grouped like Free living, Symbiotic,
Phosphorous mobilizing, P Solubilizing etc
Symbiotic Nitrogen Fixing Bacteria:
They form a mutually beneficial association with the plants. The bacteria obtain food and
shelter from plants. In return, they give a part of their fixed nitrogen to the plants. The most
important of the symbiotic nitrogen fixing bacteria is Rhizobium (pi Rhizobia). It forms
nodules on the roots of legume plants. There are about a dozen species of Rhizobium which
form association with different legume roots.
Free Living Nitrogen Fixing Bacteria:
They live freely in the soil and perform nitrogen fixation. Some of them are saprotrophic,
living on organic remains, e.g., Azoto- bacter, Bacillus polymyxa, Clostridium, Beijerinckia.
They are further distinguished into aerobic and anaerobic forms.
In case you wonder, how it is in news in recent times? See here
http://www.thehindu.com/news/cities/Coimbatore/tamil-nadu-tops-in-use-ofbiofertilizers/article8118408.ece

www.iasbaba.com

General Science and Technology- Subject wise compilation of 60 Day Plan- 2016
Also learn about differences between liquid and solid bio-fertilizers?
Q.2) Arrange the following in increasing order of their water content.
1. Egg
2. Milk
3. Meat
4. Fish
Select the correct code
a) 1-4-3-2
b) 2-1-4-3
c) 1-3-4-2
d) 4-1-3-2
Q.2) Solution (c)
NCERT- 9th Class, Chapter 15
Water Content
Milk (Cow)- 87.20
Egg - 74.00
Meat- 74.20
Fish- 77.20
Q.3) Consider the following regarding India Fishery Production
1. India fisheries is the second largest in the world
2. Inland fish production is more than marine in India
3. Contribution of fishery to Indian agriculture is less than its contribution towards GDP
Select the correct code
a) 1 and 2 only
b) Only 2
c) Only 3
d) 2 and 3 only
Q.3) Solution (b)
Again picked from class 9th NCERT- chapter 15 IMPROVEMENT IN FOOD RESOURCES FISH
PRODUCTION
Refer this link of Government Website
http://nfdb.gov.in/html/aboutus.htm
Q.4) Consider the following statements regarding Vertebrates

www.iasbaba.com

General Science and Technology- Subject wise compilation of 60 Day Plan- 2016
1. The majority of animal species are vertebrates
2. Shark is an example of vertebrate
3. Vertebrates can be identifies by the presence of notochord and gill pouches
Select the correct code
a) 1 and 2
b) 2 and 3
c) 1 and 3
d) 1, 2 and 3
Q.4) Solution (b)
Again picked from class 9th NCERT- chapter 7 IMPROVEMENT IN FOOD RESOURCES FISH
PRODUCTION
The majority of animal species are invertebrates; one estimate puts the figure at 97%.
Vertebrates make up about 4% of all described animal species; the rest are invertebrates,
which lack vertebral columns.
In case you think how vertebrate forms part of current affairs, have a look at this link
http://www.thehindu.com/news/national/kerala/high-vertebrate-diversity-recorded-instate/article7902615.ece
Q.5) Consider the following w.r.t Meristematic Tissues
1. Meristematic tissues are a group of young cells that are in a continuous state of division
2. They are living and thin walled
3. They have few vacuoles and are small in size
Select the incorrect code
a) 1, 2 and 3
b) Only 2
c) 2 and 3
d) None
Q.5) Solution (d)
Picked from class 9th NCERT- chapter 6 TISSUES
Q.6) Consider the following statements regarding Chronic Diseases
1. Some of the common examples of chronic diseases are arthritis, cardiovascular disease,
AIDS and Cancer
2. Chronic diseases can be Non-Communicable Disease (NCD)
3. Diabetes is a type of chronic disease
Select the correct code

www.iasbaba.com

General Science and Technology- Subject wise compilation of 60 Day Plan- 2016
a)
b)
c)
d)

1 and 2
2 and 3
Only 1
1, 2 and 3

Q.6) Solution (d)


Picked from class 9th NCERT- chapter 13 WHY DO WE FALL ILL
In case you wonder it is not in current affairs, click below
http://www.thehindu.com/news/cities/bangalore/not-everyone-can-afford-to-fightchronic-diseases/article8608998.ece
http://www.thehindubusinessline.com/news/blueprint-on-chronic-diseases-calls-forpreventive-care/article7829903.ece
Q.7) Recently scientists made a historic detection of Gravitational Waves. The discovery
would represent a scientific landmark, opening the door to an entirely new way to
observe the cosmos and unlock secrets about the early universe and mysterious objects
like black holes and neutron stars. Consider the following w.r.t Gravitational Waves
1. Gravitational waves can be produced by Humans, Cars and Aeroplanes.
2. The strongest gravitational waves are produced by catastrophic events such as colliding
black holes, the collapse of supernovae and the remnants of gravitational radiation
created by the birth of the Universe itself.
Select the correct code
a) Only 1
b) Only 2
c) Both
d) None
Q.7) Solution (c)
Gravitational waves are 'ripples' in the fabric of space-time caused by some of the most
violent and energetic processes in the Universe. Albert Einstein predicted the existence of
gravitational waves in 1916 in his general theory of relativity. Einstein's mathematics
showed that massive accelerating objects (such as neutron stars or black holes orbiting each
other) would disrupt space-time in such a way that 'waves' of distorted space would radiate
from the source (like the movement of waves away from a stone thrown into a pond).
Furthermore, these ripples would travel at the speed of light through the Universe, carrying
with them information about their cataclysmic origins, as well as invaluable clues to the
nature of gravity itself.
The strongest gravitational waves are produced by catastrophic events such as colliding
black holes, the collapse of stellar cores (supernovae), coalescing neutron stars or white

www.iasbaba.com

General Science and Technology- Subject wise compilation of 60 Day Plan- 2016
dwarf stars, the slightly wobbly rotation of neutron stars that are not perfect spheres, and
the remnants of gravitational radiation created by the birth of the Universe itself.
Any object with mass that accelerates (which in science means changes position at a
variable rate, and includes spinning and orbiting objects) produces gravitational waves,
including humans and cars and airplanes etc. But the gravitational waves made by us here
on Earth are much too small to detect. In fact, it isnt even remotely possible to build a
machine that can spin an object fast enough to produce a detectible gravitational wave
even the worlds strongest materials would fly apart at the rotation speeds such a machine
would require.
http://www.thehindu.com/specials/in-depth/gravitational-wavesexplained/article8224119.ece
http://www.thehindu.com/sci-tech/listening-to-the-chirps-of-gravitationalwaves/article8233625.ece
Q.8) Consider the following regarding Light Emitting Diodes (LED)
1. They work on the principle of electroluminescence
2. White LEDs are used in sterilization and hygiene since it can kill harmful bacteria
3. Blue led can be used for flat screens and date storage since it is used in compact disks to
store data.
Select the correct code
a) 1 and 2
b) 2 and 3
c) 1, 2 and 3
d) 1 and 3
Q.8) Solution (d)
The applications of blue led and white led are different. In general, white led is usually used
in lighting applications. Well, blue led can be used in sterilization and hygiene since it can kill
harmful bacteria that can cause dental and skin problems. Blue led also can be used for flat
screens and date storage since it is used in compact disks to store data.
A light-emitting diode (LED) is a two-lead semiconductor light source. It is a pn junction
diode, which emits light when activated. When a suitable voltage is applied to the leads,
electrons are able to recombine with electron holes within the device, releasing energy in
the form of photons. This effect is called electroluminescence, and the color of the light
(corresponding to the energy of the photon) is determined by the energy band gap of the
semiconductor.
http://www.thehindu.com/sci-tech/technology/internet/led-bulb-could-connect-you-tointernet/article8253782.ece

www.iasbaba.com

General Science and Technology- Subject wise compilation of 60 Day Plan- 2016
http://www.thehindu.com/business/Industry/switching-to-led-bulbs-could-save-you-morethan-rs4000-a-year/article8447542.ece
Q.9) Generic Drugs have gain lot of attention in recent times due to Indo-US spate over
IPR and Drug regime. Consider the following statements, to gain FDA approval, a generic
drug must:
1. Contain the same active ingredients as the innovator drug
2. Be identical in strength, dosage form, and route of administration
3. Be bioequivalent
4. Meet the same batch requirements for identity, strength, purity, and quality
Select the correct code
a) 1, 2 and 3
b) 2 and 4
c) 1, 3 and 4
d) 1, 2, 3 and 4
Q.9) Solution (d)
Health professionals and consumers can be assured that FDA approved generic drugs have
met the same rigid standards as the innovator drug. To gain FDA approval, a generic drug
must:
contain the same active ingredients as the innovator drug(inactive ingredients may vary)
be identical in strength, dosage form, and route of administration
have the same use indications
be bioequivalent
meet the same batch requirements for identity, strength, purity, and quality
be manufactured under the same strict standards of FDA's good manufacturing practice
regulations required for innovator products
http://www.thehindu.com/news/cities/bangalore/even-doctors-feel-generic-drugs-aresubstandard/article8398116.ece
http://www.thehindu.com/opinion/editorial/editorial-on-ban-on-generic-medicines-moremedicine-for-less/article8528990.ece
Q.10) The process of nitrogen-fixation by bacteria does not take place in the presence of
a) Molecular form of hydrogen
b) Elemental form of oxygen
c) Water
d) Elemental form of nitrogen
Q.10) Solution (b)

www.iasbaba.com

General Science and Technology- Subject wise compilation of 60 Day Plan- 2016
Picked from Class 9th Science- Chapter 14, Natural Resources
Q.11) Which of these options are not a function of Ribosomes?
1. It helps in manufacture of protein molecules
2. It helps in manufacture of enzymes
3. It helps in manufacture of hormones
4. It helps in manufacture of starch molecules
Select the correct code
a) 1 and 2
b) 2 and 3
c) 3 and 4
d) 1 and 4
Q.11) Solution (c)
Picked from Class 9th Science- Chapter 5, THE FUNDAMENTAL UNIT OF LIFE
Q.12) Which of the following are homogeneous in nature?
1. Ice
2. Wood
3. Soil
4. Air
Select the correct code
a) 1 and 3
b) 2 and 4
c) 1 and 4
d) 3 and 4
Q.12) Solution (c)
Picked from Class 9th Science- Chapter 2, IS MATTER AROUND US PURE
Q.13) Consider the following statements regarding Lifi technology:
1. Li-Fi technology uses visible light for data transfer and communication.
2. Li-Fi uses common household LED (light emitting diodes) lightbulbs to enable data
transfer, boasting speeds of up to 224 gigabits per second.
3. On-off light signals are received as binary signals at very high speed by a receptor.
Which of the above statements are correct?
a) 1 and 2
b) 2 and 3

www.iasbaba.com

General Science and Technology- Subject wise compilation of 60 Day Plan- 2016
c) 1 and 3
d) All of the above
Q.13) Solution (d)
Light Fidelity or Li-Fi is a Visible Light Communications (VLC) system running wireless
communications travelling at very high speeds.
Li-Fi uses common household LED (light emitting diodes) lightbulbs to enable data transfer,
boasting speeds of up to 224 gigabits per second.
Q.14) Rotavirus is the most common cause of severe vomiting and diarrhoea among
infants and young children. Which of the following statements are correct about rotavirus:
1. It is the biggest cause of death of children under 5 years of age in India.
2. India has recently launched an indigenous vaccine for rotavirus called Rotavac.
3. Rotavac has been added in Indias Universal Immunisation Program.
Select the correct code from the following:
a) 1 and 2
b) 2 and 3
c) 1 and 3
d) All of the above
Q.14) Solution (d)
Diarrhoea caused by Rotavirus is one of the leading causes of severe diarrhoea and death
among children less than five years of age. In India, between 80,000 to one lakh children die
due to Rotavirus diarrhoea annually while nearly 9 lakh children are admitted to hospital
with severe diarrhoea.
The Rotavac is in addition to three new vaccines that have been introduced in Indias
Universal Immunisation Programme (UIP) including Inactivated Polio Vaccine (IPV), Measles,
Rubella (MR) vaccine, and Adult Japanese Encephalitis (JE) vaccine.
Q.15) The acoustic frequencies used in sonar systems
1. ultrasonic waves
2. infrasonic waves
3. radio waves
4. audible sound waves
Select the correct code
a) 1 and 2
b) Only 1
c) 1 and 3
d) 1 and 4

www.iasbaba.com

General Science and Technology- Subject wise compilation of 60 Day Plan- 2016

Q.15) Solution (a)


The acoustic frequencies used in sonar systems vary from very low (infrasonic) to extremely
high (ultrasonic). The study of underwater sound is known as underwater acoustics or
hydroacoustics.
Concept picked from Class 9th Science- Chapter 12, SOUND
Q.16) Which of the following reactions are irreversible chemical changes:
1. Rusting of Iron
2. Cooking of food
3. Fermentation of grapes
4. Formation of ozone
Select the correct code from the following:
a) All of the above
b) 1,2 and 3
c) 2,3 and 4
d) 1,3 and 4
Q.16) Solution (b)
Formation of Ozone is a reversible process. Once an ozone molecule is formed, it breaks
again into oxygen gas(O2) and Oxygen molecule(O).
Q.17) Which of the following processes are exothermic i.e. they release energy?
1. Burning of coal
2. Respiration
3. Decomposition of vegetables to form compost
Select the code from the following:
a) 1 and 2
b) 2 and 3
c) 1 and 3
d) All of the above
Q.17) Solution (d)
In the process of respiration, the food we eat reacts with the oxygen we breathe and
releases energy. Note: all the three examples are taken from class X NCERT.
Q.18) If someone is suffering from the problem of acidity. What is the ideal thing he
should take to have relief?
a) Lemon juice

www.iasbaba.com

General Science and Technology- Subject wise compilation of 60 Day Plan- 2016
b) Vinegar
c) Baking soda solution
d) Aerated soft drink
Q.18) Solution (c)
Baking soda solution is a basic solution which helps in nutralising the acidity. Other three are
acidic which will aggravate the problem.
Q.19) Consider the following statements:
1. Distilled water does not conduct electricity.
2. The pH of distilled water is seven.
Which of the above statements are correct?
a) 1 only
b) 2 only
c) Both 1 and 2
d) Neither 1 nor 2
Q.19) solution (c)
Distilled water is pure water and does not contain any salt ions. Pure water is a bad
conductor of electricity.
Distilled water is neither acidic nor basic. The pH value is 7.
Q.20) Copper was the first metal to be used by mankind on a large scale. What was the
correct reason behind that?
a) Copper is less reactive and can be easily extracted from its ore.
b) Copper was the most abundant metal of the world.
c) Copper had higher conductivity than iron or aluminum.
d) Copper had religious significance.
Q.20) Solution (a)
Copper was accidentally discovered by man. It was extensively used as it was easy to extract
it from its ore. (Just by heating)
Q.21) Consider the following statements regarding metal alloys:
1. An alloy is a homogenous mixture of two or more metals.
2. A non-metal can also be added in an alloy with base metal.
3. An alloy has improved qualities over base metal and also prevents corrosion.
Which of the above statements are correct?
a) 1 and 2

www.iasbaba.com

10

General Science and Technology- Subject wise compilation of 60 Day Plan- 2016
b) 2 and 3
c) 1 and 3
d) All of the above
Q.21) Solution (d)
An alloy is a homogeneous mixture of two or more metals, or a metal and a non-metal. Eg.
Carbon is mixed with iron to get steel.
Q.22) Food cans are coated with tin and not with zinc because:
a) Zinc is costlier than tin.
b) Zinc has a higher melting point than tin.
c) Zinc is more reactive than tin.
d) Zinc is less reactive than tin.
Q.22) Solution (c)
Zinc can react with food items especially acidic things like vinegar, lemon juice etc. and can
contaminate the food.
Q.23) Consider the following statements:
1. Sexual reproduction leads to higher biodiversity than asexual reproduction.
2. The genetic material is transferred to the progeny in sexual reproduction and not in
asexual reproduction.
3. Asexual reproduction results in almost identical progeny.
Which of the above statements are correct?
a) 1 only
b) 2 and 3
c) 1 and 3
d) All of the above
Q.23) Solution (c)
Sexual reproduction leads to intermixing of jeans which causes more diversity.
In both sexual as well as asexual reproduction, genetic material is transferred.
Since in asexual reproduction, genetic material is copied, the progeny is almost an identical
copy.eg. binary fission of an amoeba.
Q.24) Consider the following statements regarding common eye defects:
1. Myopia is near sightedness i.e. a person is unable to clearly see nearby objects.
2. Myopia is corrected using concave lens of suitable power.
3. Hypermetropia is far sightedness i.e. a person is unable to clearly see distant object.
4. Hypermetropia is corrected using a convex lens.

www.iasbaba.com

11

General Science and Technology- Subject wise compilation of 60 Day Plan- 2016

Which of the above statements are incorrect?


a) 1 and 3
b) 2 and 4
c) None of the above
d) All of the above
Q.24) Solution (a)
1 and 3 are incorrect. Myopia is nearsightedness i.e. a person is unable to see the distant
objects. While hypermetropia is farsightedness i.e. a person is unable to clearly see the
nearby objects.
Q.25) Recently a group of researchers at Harvard University who are working on
Alzheimer disease came up with certain findings. Consider the following statements with
respect to Alzheimers disease
1. Alzheimers disease is the most common form of dementia.
2. Alzheimer is caused by Bacteria.
3. As of now there is no cure for Alzheimers
Select the correct option with respect to Alzheimers
a) 1 and 2 only
b) 2 and 3 only
c) 1 and 3 only
d) All
Q.25) Solution (c)
The exact of cause of the disease is still unknown.
Q.26) Consider the following
1. The protocol of bio safety deals with Living modified organisms
2. The bio safety protocol is also known as Cartagena protocol
3. The Nagoya and Cartagena protocol deals with Genetic resources
Identify the wrong statement with respect to Bio safety
a) 1 and 2 only
b) 2 only
c) 1 and 3 only
d) None of the above
Q.26) Solution (d)
All the statements are correct

www.iasbaba.com

12

General Science and Technology- Subject wise compilation of 60 Day Plan- 2016

Q.27) Consider the following statements in regard to vaccines:


1) BCG, or bacille Calmette-Guerin provide immunization against diarrhea given to child up
to one to four month
2) DPT refers to a class of combination vaccines against three infectious diseases in
humans: diphtheria, pertussis (whooping cough), and tetanus given to child up to one
year
3) Oral Polio vaccine (OPV) is given to the child from 2 month to 6 year
Which of the above statements is/are incorrect?
a) 2 and 3 only
b) 1 only
c) 2 only
d) 1, 2 and 3
Q.27) Solution (b)

BCG, or bacille Calmette-Guerin, is a vaccine for tuberculosis (TB) disease. Many foreignborn persons have been BCG-vaccinated. BCG is used in many countries with a high
prevalence of TB to prevent childhood tuberculous meningitis and miliary disease. (*For
diarrhea Rotavirus vaccine)
DPT (also DTP and DTwP) refers to a class of combination vaccines against three
infectious diseases in humans: diphtheria, pertussis (whooping cough), and tetanus. The
vaccine components include diphtheria and tetanus toxoids and killed whole cells of the
organism that causes pertussis (wP).
There are two types of vaccine that protect against polio: inactivated polio vaccine (IPV)
and oral polio vaccine (OPV). IPV is given as an injection in the leg or arm, depending on
the patient's age. Polio vaccine may be given at the same time as other vaccines.

Doubts: according to national immunisation schedule:


BCG: at birth or upto one year if not given earlier
OPV: at birth, 6-10-14 weeks, 16-24 months
DPT: 6-10-14 weeks, 16-24 months, 5-6 years (DT only as pertussis incidence is rare after 5...
so no point giving it)
Clarifications:
This question was framed with an intention that as a common man we should be aware
of these vaccines and against what diseases they are provided. We did not intend to
check the knowledge of duration/timings.
But thanks to some of the medical students in our IASbaba family, let us learn the
duration/schedule too.

www.iasbaba.com

13

General Science and Technology- Subject wise compilation of 60 Day Plan- 2016

www.iasbaba.com

14

General Science and Technology- Subject wise compilation of 60 Day Plan- 2016

Q.28) Which among the following are features of Mammal?


1) Sweat glands and oil glands are found on skin
2) Tooth comes twice in these animals
3) There is no nucleus in its blood cells
4) All non-egg laying animals are mammals and hence man is also a mammal
Choose the correct code
a) 1 and 4 only
b) 2 and 4 only
c) 1, 2 and 3 only
d) All the above
Q.28) Solution (c)
Mammal Characteristics
All mammals are warm blooded.
Most young are born alive.
They have hair or fur on their bodies.
Every mammal is a vertebrate.
All mammals have lungs to breathe air.
Its hearts are divided into four chambers.
Mammals feed milk to their babies.
There is no nucleus in its blood cells (except in camel and lama).
External ear is present in mammal.
Doubts: babaji pranam ... question number 12.. "
All non-egg laying animals are mammals and hence man is also a mammal " is also correct
statement .. because there are 2 types of mammals one that lays eggs (Monotremes ->
platypus and echidna) , and the ones that do not lay eggs .. so basically the latter is a subset
of the mammals group hence making the statement correct ... please throw some light and
guide me to the right path with your BABAtastic powers...
Clarification: There are some non-egg laying animals which are not mammals (hence
statement 4 is wrong)
In other words there are few fishes, few snakes and few other animals that give live births
without eggs. For example, king snakes or the rat snakes are cold blooded animals and they
dont lay egg. So we cant consider the statement All non-egg laying animals are mammals
as correct.
Q.29) Which of the following statements is/are correct?
www.iasbaba.com

15

General Science and Technology- Subject wise compilation of 60 Day Plan- 2016
1) PSLV is designed mainly to deliver the earth observation or remote- sensing
satellites in LEO
2) GSLV is designed mainly to deliver the communication- satellites in GEO
3) An orbit is called sun-synchronous when the angle between the line joining the centre of
the Earth and the satellite and the Sun is constant throughout the orbit
4) Low Earth Orbit (LEO) is in the range of 600-900 Km altitude whereas Geo-synchronous
Earth orbit (GEO) is about 36000 Km altitude
Choose the correct code
a) 1 and 2
b) 3 and 4
c) 1, 2 and 4
d) All of the above
Q.29) Solution (d)
Sun-Synchronous - circular polar orbits (The remote sensing satellites orbit the earth
from Pole-to- Pole)
GEO - zero deg inclination on equatorial plane (satellites in these orbits appear to
remain permanently fixed in the same position in the sky, as viewed from a particular
location on Earth, thus avoiding the need of a tracking ground antenna and hence are
useful for the communication applications)
Remaining are self-explanatory (as all the given options are correct)
Q.30) Match List I with List II and select the correct answer using the code given below the
Lists:
LIST I
LIST II
A. Dawn
1) Pluto
B. New Horizon
2) Ceres
C. Philae Lander
3) Binary Star
D. Cassini
4) Saturn
E. Nustar
5) comet 67P
a)
a)
b)
c)
d)

A-B-C-D-E
1-2-5-4-3
2-1-5-3-4
2-1-5-4-3
1-4-5-2-3

Q.30) Solution (c)


Dawn is a space probe launched by NASA in September 2007 with the mission of
studying two of the three known protoplanets of the asteroid belt, Vesta and Ceres. It is

www.iasbaba.com

16

General Science and Technology- Subject wise compilation of 60 Day Plan- 2016
currently in orbit about its second target, the dwarf planet Ceres. NASAs New Horizons
mission is helping us understand worlds at the edge of our solar system by making the
first reconnaissance of the dwarf planet Pluto and by venturing deeper into the distant,
mysterious Kuiper Belt a relic of solar system formation.
Philae is a robotic European Space Agency lander that accompanied the Rosetta
spacecraft until it landed on comet 67P/ChuryumovGerasimenko, more than ten years
after departing Earth.
Philae's mission was to land successfully on the surface of a comet, attach itself, and
transmit data about the comet's composition.
Cassini is an unmanned spacecraft sent to the planet Saturn. It is a flagship-class NASA
ESAASI robotic spacecraft
NASAs Nuclear Spectroscopic Telescope Array, or NuSTAR, has successfully deployed its
lengthy mast, giving it the ability to see the highest energy X-rays in our universe, a step
closer in the hunt for black holes.
Dr. Bhaleraos team observed the binary star (binary star a pair of stars revolving around
each other) using a space x-ray telescope known as NuStar, a NASA space mission. It is the
first x-ray space telescope that can focus on very high energy x-rays.
Q.31) Consider the following in regard to INO:
1) Neutrinos are colloquially called ghost particles because they travel at almost the
speed of light, hardly interact with matter, and are very light.
2) Neutrinos come in three types, and spontaneously switch from one type to another and
the process is called Neutrino Oscillation
3) In the 1960s, a neutrino observatory located at the Kolar Gold Fields in Karnataka
became one of the worlds first experiments to observe neutrinos in the Earths
atmosphere
4) At present Dept. of Atomic Energy and Dept. of Science and Technology is funding INO
to come up near Bodi Hills, Theni district, TN
Which of the above statements is/are incorrect?
a) 1 and 2
b) 2 and 3
c) None of the above
d) All of the above
Q.31) Solution (c)
Self-explanatory (All the statements given explains about Neutrinos)
Q.32) When an incandescent electric bulb glows
a) the electric energy is completely converted into light energy
b) the electric energy is partly converted into light energy and partly into heat energy

www.iasbaba.com

17

General Science and Technology- Subject wise compilation of 60 Day Plan- 2016
c) the light energy is converted into electric energy
d) the electric energy is converted into magnetic energy
Q.32) Solution (b)
When an incandescent electric bulb glows the electric energy is partly converted into light
energy and partly into heat energy
Q.33) Which one among the following statements correctly defines the term Biofortification?
a) Enrichment of the nutrient quality of the soil using biological agents
b) Breeding crops to increase their nutritional values
c) Increase of organo-metallic compounds in the organism through energy flow in an
ecosystem
d) Adoption of genetic variable plants for breeding of high yielding varieties
Q.33) Solution (b)
Biofortification is the idea of breeding crops to increase their nutritional value. This can
be done either through conventional selective breeding, or through genetic engineering.
Biofortification differs from ordinary fortification because it focuses on making plant
foods more nutritious as the plants are growing, rather than having nutrients added to
the foods when they are being processed.
Q.34) 3G and 4G are wireless technologies, which are supposedly faster, more secure and
reliable. Present day 3G technology is capable of handling data around 2 Megabits per
second (1.8 - 2.5 GHz frequency band). What speed is expected from new 4G technology?
a) 10-30 Megabits per second (1-5 GHz frequency band)
b) 10-100 Megabits per second (2-10 GHz frequency band)
c) 100 Megabits - 1 Gigabit per second (2-8 GHz frequency band)
d) More than 10 Gigabits per second (10-20 GHz frequency band)

www.iasbaba.com

18

General Science and Technology- Subject wise compilation of 60 Day Plan- 2016
Q.34) Solution (c)
3G should be capable of handling around 2 Megabits per second.
4G The speed and standards of this technology of wireless needs to be at least 100
Megabits per second and up to 1 Gigabit per second to pass as 4G.
Q.35) Which among the following is/are diseases caused by Protozoa?
1) Malaria
2) Diarrhea
3) Kala-azar
4) Sleeping sickness
Choose the appropriate code:
a) 4 only
b) 1 and 3 only
c) 2 and 4 only
d) All of the above
Q.35) Solution (d)
All the given diseases are caused by Protozoa
Protozoan infections are parasitic diseases caused by organisms formerly classified in
the Kingdom Protozoa.
Protozoa are single-celled micro-organisms that get their food from the surrounding
environment or as scientists would say 'they are heterotrophs. They can be found in a
variety of habitats such as freshwater (including ponds & rivers), marine (the sea) and
even in the soil.
Q.36) Match List I with List II and select the correct answer using the code given below the
Lists:
List I List II
A. Vitamin A
1.Non-clotting of blood
B. Vitamin B
2.Rickets
C. Vitamin C
3.Beriberi
D. Vitamin D
4.Colour Blindness
E. Vitamin K
5.Scurvy
A-B-C-D-E
a) 4-3-5-2-1
b) 4-1-5-2-3
c) 4-1-5-3-2
d) 2-1-5-4-3

www.iasbaba.com

19

General Science and Technology- Subject wise compilation of 60 Day Plan- 2016
Q.37) Solution (a)
Doubts: baba ji vitamin A causes night blindness not colour blindness because colour
blindness is a sex linked trait.
Colour Blindness is genetic..... Yeah, I think it was a typo, and they meant to write Night
Blindness
Clarification:
What causes colour blindness?
The vast majority of colour vision problems are inherited (genetic) and are present at birth.
However, other much less common causes include shaken baby syndrome, accidents and
other trauma which may cause swelling of the brain in the occipital lobe, as well as damage
to the retina caused by exposure to ultraviolet light. Sometimes, colour blindness caused by
damage to the areas of the brain where vision processing takes place doesnt become
apparent until later in life.
Further causes of colour blindness are a deficiency of vitamin A and degenerative diseases
of the eye macular degeneration, glaucoma, and retinal damage from diabetes as well as
cataracts.
Refer: http://www.ncbi.nlm.nih.gov/pmc/articles/PMC1604036/?page=1
Q.38) DNA Origami involves making shapes out of DNA in
a) 1 Dimension
b) 2 Dimension
c) 1 and 2 Dimensions
d) 2 and 3 Dimensions
Q.38) Solution (d)
http://www.thehindu.com/opinion/op-ed/what-is-dna-origami/article8660871.ece
Q.39) Consider the following
1. Change in state
2. Change in temperature
3. Change in colour
4. Evolution of a gas
Which of the above observations leads to a chemical reaction?
a) 1, 3 and 4
b) 2, 3 and 4
c) 1, 2 and 3
d) 1, 2, 3 and 4
Q.39) Solution (d)
Refer to NCERT, Chemical Reactions and Equations

www.iasbaba.com

20

General Science and Technology- Subject wise compilation of 60 Day Plan- 2016
Doubt- Hi sir, wrt Q2. i think the question statement was inadequate. the statement in book
"the following observations helps us to determine whether a chemical reaction has taken
place" is reference to a particular experiment carried out in the book. This however cannot
be generalized as "observations leads to a chemical reaction?". clearly transformation of ice
to water involves change in state without any chemical reaction.
Clarification: Question says leads to chemical reaction. And just because transformation
of ice to water doesnt involve any chemical change it doesnt mean its wrong
Always focus on general interpretation without going deep. Unless it is governed by words
like Only, Always etc
A chemical reaction is a process that leads to the transformation of one set of chemical
substances to another.
Change in State
Solid melts to liquid or vaporizes to gas. Change in state as well as formation of different
chemical compounds. Remember displacement or synthesis reactions. Refer the below
reaction
Mg (solid) + 2HCl (aqueous) MgCl2 (aqueous) + H2 (gas)
Change in Temperature
Recall about exothermic and endothermic change that brings about chemical reaction or
change
Change in Color
Majority of organometallic compounds and basic chemistry tests are based on identification
of change in color. Remember doing experiment of Copper Sulphate in school
Evolution of Gas
Obviously it is. Remember decomposition reaction
So answer should be (d)
Q.40) Consider the following
1. Fuelling of Space Shuttle
2. Photosynthesis
3. Corrosion of metals
4. Rancidity
5. Enzymes
6. Combustion
Which of the above involves Redox reaction?
a) 1, 2 and 5
b) 2, 3, 5 and 6
c) 1, 2, 4 and 5

www.iasbaba.com

21

General Science and Technology- Subject wise compilation of 60 Day Plan- 2016
d) 1, 2, 3, 4, 5 and 6
Q.40) Solution (a)
Combustion, Rancidity and Corrosion: Oxidation Reaction.
Doubt- --corrosion, combustion and rancidity are common examples of redox reactions
from everyday life..free radicals are involved in rancidity.. so redox..
Clarification:
Redox reactions are not always governed by free radicals or ions. A simple oxidation and
reduction taking place simultaneously is also a Redox reaction.
Keep in mind that this question doesnt require you to go deep and dig into free radical
concepts
Think like this!
Redox- Oxidation and Reduction simultaneously (Reversibly)
Photosynthesis, Fuelling of space shuttle (the internal reaction) and Enzyme activity are
reversible process i. e Oxidation <=>Reduction taking place on its own in a system
Whereas corrosion, rancidity and combustion are irreversible reactions.
Once a metal get corroded it wont come to normal unless external adjustments are done
(Reducing agents are used).
Conclusion: Since there was no specification given in the question, both the arguments as
asked in the doubt and as explained here holds true. So the best choice should be (d)
More in brief
The reduction and oxidation reaction or redox reaction is a chemical reaction during which
the oxidation state of two or more of the reactants change. Oxidation reaction is not only
about just combining with oxygen. Most redox reactions do not even involve oxygen at all.
Oxidation in the context of a redox reaction refers to the oxidation state of an atom being
increased.
Conversely reduction refers to oxidation state of an atom being reduced.
Many chemical reactions do not involve any change in oxidation state.
For example when we react aqueous solutions of copper (II) sulphate and sodium hydroxide
to form copper (II) hydroxide which precipitates and a solution of sodium sulphate forms
and there are no changes in oxidation state.
Oxidation is the process of addition of oxygen or any electronegative radical or removal of
hydrogen or electropositive radical. Its a process in which an atom or a group of atoms
taking part in a chemical reaction loses one or more electrons.
The species which undergo the loss of electron during the reaction is called as reducing
agent or reductant.

www.iasbaba.com

22

General Science and Technology- Subject wise compilation of 60 Day Plan- 2016
An oxidation reaction is always couple with reduction reaction which refers as addition of
electron to the reaction species (Oxidant or oxidising agent).
These coupled reactions are known as redox reaction.
We can observe many oxidation reactions in our daily life like corrosion of metal, rancidity
and combustion.
Combustion: it is the most common example of oxidation reaction.
Combustion or burning of any material involves oxidation reaction coupled with reduction.
The complete combustion of substance generally released carbon dioxide and water.
For example the burning of wood released a large amount of energy with carbon dioxide
and water vapour.
That energy uses for heating home, drive automobile, operate industrial processes and
much other purpose.
Corrosion: You must have seen the rusting of your car and the burning of magnesium metal
in oxygen to form magnesium oxide in your daily life are also an oxidation reaction.
Similarly rusted iron sheets or green surface of copper utensils and tableware or tarnish
surface of aluminium surfaces are also due to oxidation of metal surface.
Most of metal surfaces oxidised due to atmospheric oxygen and forms metal oxides on the
surface of metal.
For example; corrosion of iron forms iron oxide which is also called as rust.
4Fe +3O2 ==> 2Fe2O3
Rancidity: Oxidation reactions are also responsible for the spoiling of food.
Oxidation-reduction reactions also fuel the most advanced form of transportation known
today, the space shuttle. The actual orbiter vehicle is relatively small compared to its
external power apparatus, which consists of two solid rocket boosters on either side, along
with an external fuel tank.
Inside the solid rocket boosters are ammonium perchlorate (NH4ClO4) and powdered
aluminium, which undergo an oxidation-reduction reaction that gives the shuttle enormous
amounts of extra thrust. As for the larger single external fuel tank, this contains the gases
that power the rocket: hydrogen and oxygen.
Because these two are extremely explosive, they must be kept in separate compartments.
When they react, they form water, of course, but in doing so, they also release vast
quantities of energy. The chemical equation for this is: 2H 2 + O 2 2H 2 O + energy.
On January 28, 1986, something went terribly wrong with this arrangement on the space
shuttle Challenger. Cold weather had fatigued the O-rings that sealed the hydrogen and
oxygen compartments, and the gases fed straight into the flames behind the shuttle itself.
This produced a powerful and uncontrolled oxidation-reduction reaction, an explosion that
took the lives of all seven astronauts aboard the shuttle.

www.iasbaba.com

23

General Science and Technology- Subject wise compilation of 60 Day Plan- 2016
Enzymes in the human body regulate oxidation-reduction reactions. These complex
proteins, of which several hundred are known, act as catalysts, speeding up chemical
processes in the body. Oxidation-reduction reactions also take place in the metabolism of
food for energy, with substances in the food broken down into components the body can
use.
Photosynthesis involves the reduction of carbon dioxide into sugars and the oxidation of
water into molecular oxygen.
Q.41) Consider the following statements
1. All non-metals are either solid or gases
2. All metals are solid at room temperature
3. All non-metals are non-lustrous
4. All non-metals are bad conductors of electricity
Select the incorrect code
a) 1, 3 and 4
b) 1, 2 and 3
c) 2 and 4 Only
d) 1, 2, 3 and 4
Q.41) Solution (d)
Bromine is a non-metal which is liquid
Mercury is a metal but exists as liquid at room temperature.
Iodine is a non-metal but lustrous
Graphite is a non-metal (allotrope of carbon) but good conductor of electricity
Q.42) Consider the following
1. Sodium
2. Gold
3. Magnesium
4. Copper
5. Calcium
6. Platinum
Which of the above are found in nature as Free Elements?
a) 1, 2 and 6
b) 2, 3 and 5
c) 2, 4 and 6
d) 1, 2, 4, 5 and 6

www.iasbaba.com

24

General Science and Technology- Subject wise compilation of 60 Day Plan- 2016
Q.42) Solution (c)
Some metals are found in Free State and some in the form of their compounds. Reactive
elements like Sodium, Potassium, Magnesium and Calcium are not found in nature as free
elements.
Q.43) Choose the incorrect statement
a) When a solid melts its temperature changes
b) Water vapour at 100 degree Celsius have more energy than normal water at the same
temperature
c) Change in pressure can change the state of matter
d) Both (a) and (b)
Q.43) Solution (a)
When a solid melts, its temperature doesnt change. The heat energy is used by the particles
to overcome the force of attraction.
http://www.dummies.com/how-to/content/why-temperature-remains-constant-during-aphase-ch.html
Q.44). Recently NASAs new horizon had sent close-up images of Pluto. Consider following
statements about New Horizons
1. New Horizons is an interplanetary space probe that was launched as a part of NASA's
New Frontiers program
2. A space probe is a robotic spacecraft that leaves Earth orbit and explores space.
3. Space environment around Pluto and its moons has a lot of dust.
Select the correct statements
a) 1 only
b) 1 and 2 only
c) All of the above
d) None of the above
Q.44) Solution (b)
Space environment around Pluto and its moons is almost empty, containing only about six
dust particles per cubic mile, according to data collected by a student-built instrument riding
on NASAs New Horizons spacecraft.
Further reading - http://www.thehindu.com/sci-tech/science/space-around-pluto-nearlydust-freenasa/article8370579.ece
Further reading - https://en.wikipedia.org/wiki/New_Horizons
Space probe may approach the Moon; enter interplanetary space; flyby, orbit, or land on
other planetary bodies; or approach interstellar space.

www.iasbaba.com

25

General Science and Technology- Subject wise compilation of 60 Day Plan- 2016
Q.45) Consider the following statements about jet lag
1. It is a physiological condition which results from alterations to the body's circadian
rhythms resulting from rapid long-distance trans-meridian travel on high-speed aircraft.
2. It is also known as desynchronosis or circadian dysrhythmia
Select the correct statements
a) 1 only
b) 2 only
c) Both 1 and 2
d) Neither 1 nor 2
Q.45) Solution (c)
Mental health implication - Jet lag may affect the mental health of vulnerable individuals.
When travelling across time zones, there is a "phase-shift of body temperature, rapid-eyemovement sleep, melatonin production, and other circadian rhythms
More reading - https://en.wikipedia.org/wiki/Jet_lag
http://www.thehindu.com/sci-tech/health/fresh-method-developed-to-deal-withjetlag/article8668714.ece
Q.46) Mentha also known as mint is a very beneficial crop for farmers in India. Consider
the statements regarding crop Mentha
1. India is a net importer of mentha
2. It is included in Minimum Support Price (MSP)
3. Menthol is obtained from plant mentha
Select the correct statement/s
a) Only 1
b) 1 and 2
c) 1, 2 and 3
d) Only 3
Q.46) Solution (d)
Just as rice, wheat and cotton are among the 25 crops that qualify for a governmentmandated minimum support price, the CSIR is framing a proposal to push for mentha (the
plant that gives menthol), lavender, lemongrass, artemisia annua (the source of the
antimalarial compound artemisinin), geranium, lavender and rose oil to be included in such
a category.
India was an importer of mentha 40 years ago. Today it is the worlds leading producer. This
is because mentha varieties were specifically bred to exude greater quantities of menthol
mint oil. In 2014, mentha acreage spanned 3,00,000 hectares.

www.iasbaba.com

26

General Science and Technology- Subject wise compilation of 60 Day Plan- 2016
http://www.thehindu.com/todays-paper/tp-national/csir-wants-farmers-to-consideralternatives/article8669484.ece
Q.47) Consider the following statements regarding e-waste
1. Pyro metallurgy and hydro metallurgy are the processes used to extract gold from ewaste
2. Pyro metallurgy is a process in which leaching chemicals are used.
3. Hydro metallurgy is a process to extract gold using high temperatures.
Select the incorrect statements
a) Only 1
b) Only 1 and 2
c) Only 2 and 3
d) All of the above
Q.47) Solution (c)
Pyro metallurgy is a process to extract gold using high temperature and hydro metallurgy is
a process to in which leaching chemicals are used.
Q.48) Consider the following statements:
1. Orbital speed of a satellite is independent of its mass
2. Orbital speed of a satellite depends upon the radius of orbit (height of satellite from the
surface of earth)
3. Period of revolution of a satellite is independent of its mass
4. Escape velocity is independent of the mass, shape and size of the body and its direction
of projection
Which of the statements given above is/are correct?
a) 3 and 4 only
b) 1, 2 and 3 only
c) 1, 3 and 4 only
d) All of the above
Q.48) Solution (d)
Orbital speed of a satellite is independent of its mass. Hence satellites of different
masses revolving in the orbit of same radius have same orbital speed.
Orbital speed of a satellite depends upon the radius of orbit (height of satellite from the
surface of earth). Greater the radius of orbit, lesser will be the orbital speed.
The orbital speed of a satellite revolving near the surface of earth is 7.9 km/sec
Time taken by a satellite to complete one revolution in its orbit is called its period of
revolution.

www.iasbaba.com

27

General Science and Technology- Subject wise compilation of 60 Day Plan- 2016

Period of revolution of a satellite depends upon the height of satellite from the surface
of earth. Greater the height more will be the period of revolution.
However, period of revolution of a satellite is independent of its mass.
The period of revolution of satellite revolving near the surface of earth is 1 hour 24
minute (84 minute)

Doubts: answer to Q.No. 11 is incorrectly shown in . it correct option is B as escape velocity


is dependent on mass etc mention in q
Clarification:

Also refer: https://books.google.co.in/books?id=yMHHWx36vOwC&pg=SA6-PA6&lpg=SA6PA6&dq=Escape+velocity+is+independent+of+the+mass,+shape+and+size+of+the+body+an


d+its+direction+of+projection&source=bl&ots=XEVfuVTyqQ&sig=itUg_g4kRBlL8wBVTGnli_W1Io&hl=en&sa=X&ved=0ahUKE
(Or) http://schools.aglasem.com/46507
Q.49) Consider the following statements and select the incorrect statement/s from the
codes given below:
1. The addition of detergent or soap decreases the viscosity of water and thus increases
the cleaning ability
2. If a clean and dry needle is kept slowly on the surface of water, it floats due to surface
tension of water
Choose the appropriate code:
a) 1 only
b) 2 only
c) Both 1 and 2
d) Neither 1 nor 2
Q.49) Solution (a)
Both the given statements are examples of surface tension property of water

www.iasbaba.com

28

General Science and Technology- Subject wise compilation of 60 Day Plan- 2016

If a clean and dry needle is kept slowly on the surface of water, it floats due to surface
tension of water
The addition of detergent or soap decreases the surface tension of water and thus
increases the cleaning ability
Surface Tension is the property of a liquid by virtue of which it has the tendency to have
the area of its free surface minimum as if it were under tension like a stretched elastic
membrane
A liquid drop attains spherical shape due to surface tension as for given volume, sphere
has minimum surface area

Q.50) Identify the incorrect pair/s from the following:


1. Biggest Solar Observatory : : Kodaikanal
2. National Remote Sensing Centre : : Mahendragiri
3. INSAT Master Control Facility : : Hassan
4. Indian Deep Space Network (IDSN) : : Byalalu
Choose the correct code:
a) 3 only
b) 3 and 4 only
c) 1 and 2 only
d) 2 only
Q.50) Solution (c)
Explanation:
Biggest Solar Observatory is in Udaipur, Rajasthan on an island in the Fateh Sagar Lake.
The sky conditions at Udaipur are quite favourable for solar observations. Since the
observatory is situated amidst a large mass of water, air turbulence which occurs due to
ground heating by sun's rays is decreased. This improves the image quality and accuracy
(average between 1-2 arc seconds).
National Remote Sensing Centre is in Hyderabad not Mahendragiri
Refer the below fig. for other space related centres
Q.51) Which among the following waves are examples of Electromagnetic waves?
1) X-rays
2) -rays
3) -rays
4) -rays
5) Ultrasonic wave
Choose the appropriate code:
a) 1, 2, 3 and 4 only
www.iasbaba.com

29

General Science and Technology- Subject wise compilation of 60 Day Plan- 2016
b) 1 and 2 only
c) 1 and 5 only
d) 1, 2 and 5 only
Q.51) Solution (b)
Electromagnetic waves or non-mechanical waves are those waves which do not require
medium for their propagation i.e. which can propagate even through the vacuum are
called non-mechanical wave.
In other words, electromagnetic waves are waves which can travel through the vacuum
of outer space. Mechanical waves, unlike electromagnetic waves, require the presence
of a material medium in order to transport their energy from one location to another.
Following waves are not electro-magnetic 1) Cathode rays 2) -rays 3) -rays 4) Sound
wave 5) Ultrasonic wave 6) Canal rays
Some important Electromagnetic waves
-rays 2) X-rays 3) UV rays 4) Visible
radiation 5) Infrared rays 6) Radio waves
Q.52) Consider the following statements in regard to Indias 3-stage Nuclear Power
Programme:
1) The first stage Pressurised Heavy Water Reactors (PHWRs) use natural uranium as fuel
and heavy water as moderator and coolant
2) The second stage Fast Breeder Reactors (FBRs) use plutonium (which is produced by
irradiation of uranium-238) and liquid sodium as coolant
3) The third stage is based on the thorium-uranium-233 cycle. Uranium-233 is obtained by
irradiation of thorium.
Which of the statements given above is/are correct?
a) 2 and 3 only
b) 1 and 2 only
c) 1 and 3 only
d) All of the above
Q.52) Solution (d)
All the statements are correct and self-explanatory
Q.53) Match List I with List II and select the correct answer using the code given below the
Lists:
LIST I LIST II
A. Anupam-Ameya 1. BARC
B. PARAM Yuva 2. ISRO
C. SAGA 3. C-DAC
D. Vikram-100 4. Physical Research Laboratory

www.iasbaba.com

30

General Science and Technology- Subject wise compilation of 60 Day Plan- 2016

A-B-C-D
a) 1-2-3-4
b) 1-3-2-4
c) 4-2-3-1
d) 4-3-2-1
Q.53) Solution (b)
VIKRAM-100 is the 13th fastest supercomputer in India and is more powerful than 200
desktop computers
VIKRAM -100 was inaugurated on 26 June 2015, by Prof. U. R. Rao at the Physical
Research Laboratory
Vikram-100 is a High Performance Computing (HPC) Cluster (named after eminent
scientist Dr Vikram Sarabhai)
Physical Research Laboratory (PRL), is an organisation that was founded by him at
Ahmedabad in 1947
Q.54) Which of the following are illustrations of total internal reflection?
1) Shining of air bubble in water
2) Sparkling of diamond
3) Increase in duration of suns visibility
4) Mirage and looming
5) Ophthalmoscope
Choose the appropriate code:
a) 1, 2, 3 and 4 only
b) 1 , 2 and 5 only
c) 1, 2, 3 and 5 only
d) All of the above
Q.54) Solution (d)
Total Internal Reflection:
If light is propagating from denser medium towards the rarer medium and angle of
incidence is more than critical angle, then the light incident on the boundary is reflected
back in the denser medium, obeying the laws of reflection. This phenomenon is called
total internal reflection as total light energy is reflected, no part is absorbed or
transmitted.
For total internal reflection,
1) Light must be propagating from denser to rarer medium
2) Angle of incidence must exceed the critical angle

www.iasbaba.com

31

General Science and Technology- Subject wise compilation of 60 Day Plan- 2016
Q.55) Which among the following statements is/are true in regard to DNA and RNA?
1) Sugar is deoxyribose type in DNA, whereas sugar is just ribose type in RNA
2) DNA is double stranded structure, whereas RNA is single stranded structure
3) DNA is found in both nucleus and cytoplasm, RNA is found only in nucleus
Which of the statements given above is/are correct?
a) 2 and 3 only
b) 1 and 2 only
c) 1 and 3 only
d) All of the above
Q.55) Solution (b)
DNA is found mainly in nucleus, whereas RNA is found in both nucleus and cytoplasm
Q.56) Which among the following statements is/are true?
1) Apart from blood groups O, A, B and AB, there is a special type of blood group (hh)
2) A type blood can accept blood from A type or O type and not from AB or B type
donors
3) The main reason behind the difference in blood of human is the glyco protein which is
found in RBC called antigen
Which of the statements given above is/are correct?
a) 2 and 3 only
b) 1 and 2 only
c) 1 and 3 only
d) All of the above
Q.56) Solution (d)
Apart from blood groups O, A, B and AB, there is a special type of blood group (hh)
(hh)- a rare special blood group first discovered in Bombay in 1952, and hence
christened as Bombay Blood.
People who carry this rare blood type, about 1 in 10, 000 Indians, can accept blood only
from another Bombay Blood type individual, and not from anyone who is O, A, B or AB
type.
Q.57) Consider the following statements with regard to Atomic Energy Regulatory Board
(AERB)
1) AERB is engaged in the development of nuclear power technology, applications of
radiation technologies in the fields of agriculture, medicine, industry, and basic research.

www.iasbaba.com

32

General Science and Technology- Subject wise compilation of 60 Day Plan- 2016
2) The safety standards formulated by AERB are at par with those recommended by the
international organisations such as the International Atomic Energy Agency (IAEA) and
the International Commission on Radiological Protection (ICRP).
Choose the appropriate code from below:
a) Only 1 is correct
b) Only 2 is correct
c) Both 1 and 2 are correct
d) Both 1 and 2 are incorrect
Q.57) Solution (b)
Department of Atomic Energy (not AERB), established in 1954 is engaged in the
development of nuclear power technology, applications of radiation technologies in the
fields of agriculture, medicine, industry, and basic research.
Atomic Energy Regulatory Board (AERB) is an independent body, the Atomic Energy
Regulatory Board (AERB) monitors safety.
The safety standards formulated by AERB are at par with those recommended by the
international organisations such as the International Atomic Energy Agency (IAEA) and
the International Commission on Radiological Protection (ICRP).
Q.58) A fuel cell is a device that generates electricity by a chemical reaction. Consider the
following
1. Molten carbonate fuel cells
2. Phosphoric Acid fuel cells
3. Hydrogen fuel cells
4. Solid oxide fuel cells
Which of the above is/are types of fuel cells?
a) Only 3
b) 2 and 3
c) 2, 3 and 4
d) 1, 2, 3 and 4
Q.58) Solution (d)
Similar to batteries and all other electrochemical cells, fuel cells have two electrodes, and an
electrolyte sandwiched in between. Fuel cells are distinguished by the type of electrolyte
they use. The more common fuel cells on the market are:
PEM Polymer Exchange Membrane also known as Polymer Electrolyte Membrane
Fuel Cell (PEMFC)
Solid Oxide Fuel cell (SOFC),
Molten Carbonate Fuel cell (MCFC),

www.iasbaba.com

33

General Science and Technology- Subject wise compilation of 60 Day Plan- 2016

Alkaline Fuel Cell (MFC),


Phosphoric Acid fuel cell (PAFC).
Direct Methanol fuel cell (DMFC) is the only type of fuel cell that is distinguished by its
fuel (Methanol) and not its electrolyte.
http://www.thehindubusinessline.com/specials/clean-tech/change-is-in-theair/article7912910.ece
http://www.thehindu.com/features/metroplus/Motoring/honda-begins-sale-of-zeroemissions-car-the-clarity-fuel-cell/article8336475.ece
Read this to know the working of fuel cells
http://www.fuelcellenergy.com/why-fuelcell-energy/how-do-fuel-cells-work/
Q.59) Which of the following is the correct definition of Transcriptome?
a) It includes only those RNA molecules found in a specified cell population, and usually
includes the amount or concentration of each RNA molecule in addition to the molecular
identities.
b) It is the set of all messenger RNA molecules in one cell or a population of cells.
c) It is the set of all transfer RNA molecule in two or more cells
d) None of the above is correct
Q.59) Solution (b)
Reference taken from- http://www.thehindu.com/news/national/karnataka/genome-ofthe-asian-elephant-sequenced-for-the-first-time-in-india/article8006026.ece
Q.60) Digital convergence refers to the convergence of four industries into one
conglomerate. Identify those four industries?
1. Media
2. Consumer Electronics
3. Information Technology
4. Telecommunication
5. Entertainment
6. Cognitive Science
Select the appropriate code
a) 1, 3, 4 and 5
b) 2, 3, 4 and 5
c) 2, 3, 4 and 6
d) 3, 4, 5 and 6
Q.60) Solution (b)

www.iasbaba.com

34

General Science and Technology- Subject wise compilation of 60 Day Plan- 2016
Digital convergence refers to the convergence of four industries into one conglomerate,
ITTCE (Information Technologies, Telecommunication, Consumer Electronics, and
Entertainment).
Q.61) Consider the following about totipotent, pluripotent and multipotent cells:
1. The correct order from most differentiated to least differentiated
pluripotent>multipotent>totipotent
2. Embryonic stem cells are considered pluripotent cells
3. Multipotent cells are more limited than pluripotent cells

is

Select the correct code


a) 1 and 2
b) 2 and 3
c) 1, 2 and 3
d) 1 and 3
Q.61) Solution (b)
These terms refer to the differentiating potential of the particular cell in its development
stage. In general, going from least differentiated to most differentiated, you could arrange
the terms as totipotent=>pluripotent=>multipotent. Now this means that a totipotent cell is
not differentiated at all, and can create any cell type in the body and certain placental
tissues. Only embryonic cells in very early stages of development (the first few cell divisions
from a zygote or fertilized ovum) are totipotent. A pluripotent stem cell can create cells
from any of the three germ layers in the body, thus the least differentiated pluripotent cells
can become any cell in the body. Embryonic stem cells are one source of pluripotent cells.
Totipotent cells can form all the cell types in a body, plus the extraembryonic, or placental,
cells. Embryonic cells within the first couple of cell divisions after fertilization are the only
cells that are totipotent. Pluripotent cells can give rise to all of the cell types that make up
the body; embryonic stem cells are considered pluripotent. Multipotent cells can develop
into more than one cell type, but are more limited than pluripotent cells; adult stem cells
and cord blood stem cells are considered multipotent
Reference of pluripotent cells taken from this news
http://www.thehindu.com/sci-tech/science/crisprcas-method-of-gene-editing-causescrisper-debates/article8203123.ece
Also learn about stem cells and induced pluripotent stem cells
Q.62) Since the advent of the antibiotic superdrug in the 1940s, a new villain, the
superbug has emerged and has been making national and international headlines over
the past two decades. Consider the following
1. Methicillin-resistant Staphylococcus aureus (MRSA)
2. Drug-resistant tuberculosis (MDR- and XDR-TB)

www.iasbaba.com

35

General Science and Technology- Subject wise compilation of 60 Day Plan- 2016
3. Drug-resistant Enterococcus
4. Drug-resistant Streptococcus pneumoniae
Which of the above falls under the category of Superbug?
a) 1, 3 and 4
b) 2, 3 and 4
c) 1, 2 and 3
d) 1, 2, 3 and 4
Q.62) Solution (d)
Reference taken from here- http://www.thehindu.com/opinion/op-ed/the-battle-againstsuperbugs/article8660875.ece?utm_source=RSS_Feed&utm_medium=RSS&utm_campaign=
RSS_Syndication
Q.63) Consider the following regarding Uranium production in the world
1. India is among top 5 uranium producing countries in the world
2. Kazakhstan produces the largest share of uranium from mines followed by Australia and
Canada
3. Worlds maximum uranium production comes from underground mining
Select the incorrect code
a) Only 1
b) 1 and 3
c) 1, 2 and 3
d) None
Q.63) Solution (c)
India is not even in top 10. Kazakhstan- Canada-Australia
Over two-thirds of the world's production of uranium from mines is from Kazakhstan,
Canada and Australia. An increasing amount of uranium, now 48%, is produced by in situ
leaching.
Reference taken from here- http://www.thehindu.com/data/nuclear-energy-a-globalfactsheet/article8701264.ece
Q.64) Why does the Sun appear white in colour at noon?
a) A little of the blue colour is scattered.
b) Blue colour is scattered the most
c) Red colour is scattered the most
d) All the colours are scattered away
Q.64) Solution (a)

www.iasbaba.com

36

General Science and Technology- Subject wise compilation of 60 Day Plan- 2016
At noon, the sun is overhead and the light coming from the sun travels a relatively shorter
distance through the atmosphere to reach the earth. In this case the blue light is not
sacttered much. As the light coming from the overhead sun contains almost all its
component colours in the right proportion, the sun appears white to us at noon.
NCERT- NCERT- class 10th
Q.65) Consider the following regarding The Natural Resources Data Management System
(NRDMS) Programme
1. It was launched with the objectives of developing and demonstrating scientific methods
and techniques for national level planning
2. Ministry of Environment and Forest is the nodal agency to govern its function and
proper execution
Select the correct code
a) Only 1
b) Only 2
c) Both
d) None
Q.65) Solution (d)
http://pib.nic.in/newsite/PrintRelease.aspx?relid=142346
Q.66) Consider the following regarding sunrise and sunset
1. The time difference between actual sunrise and sunset is 4 minutes
2. This is because of the phenomenon of Total Internal Reflection
3. The apparent flattening of the Suns disc at sunrise and sunset is due to atmospheric
refraction.
Select the incorrect code
a) Only 1
b) Only 3
c) 1 and 2
d) 1, 2 and 3
Q.66) Solution (c)
Time difference is 2 minutes and it is due to atmospheric refraction
A ray of light bends when it travels from one medium to another. This bending of light is
called refraction of light. If the light ray travels from a rare medium to a denser medium, it
bends towards the imaginary normal and if it travels from a dense medium to a rarer
medium, it bends away from the imaginary normal. Depending on the density of the

www.iasbaba.com

37

General Science and Technology- Subject wise compilation of 60 Day Plan- 2016
different medium, the speed of the travelling light ray keeps varying, and this causes it to
slow down or speed up, therefore bending in the process.
So how this refraction of light is connected to our advanced sunrise and delayed sunset?
Imagine the journey of light rays from the sun. Their initial journey is through vacuum and
then through the atmosphere of the earth and then it is finally seen by us. In this case,
vacuum will be a rare medium and the earths atmosphere with all its temperature changes,
winds, different gases, will be a denser medium in comparison.
During sunrise, when the sun is just below the horizon, our atmosphere causes the light rays
to bend and we see the sun early. Similarly, at sunset, the apparent position of the sun is
visible to us and not the actual position due to the same bending of light rays effect.
To sum up, due to refraction we see the sun rise about two minutes before its actually
there and during sunset we see it for around two minutes more, even though it has already
moved from that position.
Q.67) Acid attacks are one of the most gruesome kind of acts causing not only physical
harm to the victim but psychological and emotional harm as well. Which of the following
steps should be taken to reduce the effect of acid on the skin?
1. Burnt area should be washed with clean water or slightly saline water.
2. Burnt are should be washed with milk. Milk is basic and neutralizes acid.
3. Burnt area should be washed with a strong base to quickly neutralize it so that it does
not react further.
Select the code from the following:
a) 1 only
b) 1 and 2
c) 1 and 3
d) All of the above
Q.67) Solution (a)
Milk is slightly acidic and also the reaction of milk and acid is exothermic which can cause
further damage. It can also contaminate the burnt area.
Strong base is itself a corrosive agent. And the reaction with acid is highly exothermic.
Q.68) Mitochondria is an organelle found in most of the cells. It is known as the
powerhouse of the cell. Which of the following statements are correct related to
mitochondria?
1. The biochemical processes of respiration and energy production occurs here.
2. Mitochondrial disease can pass to a progeny only through mothers in human beings.
3. Three parent baby is a technique used to prevent the transfer of mitochondrial defect to
the babies.

www.iasbaba.com

38

General Science and Technology- Subject wise compilation of 60 Day Plan- 2016
Select the correct code from the following:
a) 1 and 2
b) 2 and 3
c) 1 and 3
d) All of the above
Q.68) Solution (d)
In a male germ cell (sperm), the tail part contains the mitochondria. As tale does not take
part in the process of fertilization, it is not passed to a progeny from fathers.
Mitochondria is present in the cytoplasm of the mothers cells. In three parent baby, the
nucleus from mothers egg cell is extracted and planted in the cytoplasm of another
womens egg cell that does not have a mitochondrial defect. This new cell is fertilized by
fathers sperm. Since the genetic characteristics lie in the nucleus, the baby gets his genetic
traits from two parents only.
Q.69) Which of the following forces are not the example of contact forces?
1. Friction force
2. Electrostatic force
3. Magnetic force
4. Gravitational force
5. Air resistance force
Select the code from the following:
a) 1 only
b) 1, 3, 4 and 5
c) 2, 3 and 4
d) 2, 4 and 5
Q.69) Solution (c)
Friction force is an example of a contact force. Rest of the three forces acts from a distance
and dont require body to body contact.
Contact Forces
Action-at-a-Distance Forces
Frictional Force
Gravitational Force
Tension Force
Electrical Force
Normal Force
Magnetic Force
Air Resistance Force
Applied Force
Spring Force
Q.70) Pole star is amongst the most useful stars for humans because of its fixed location in
the sky. Consider the following statements with respect to pole star:

www.iasbaba.com

39

General Science and Technology- Subject wise compilation of 60 Day Plan- 2016
1. Pole star is helpful in finding the north direction in night and helped in navigation in
ancient times.
2. Pole star can be pointed with the help of Ursa Major constellation.
3. Pole star is not visible in the Southern Hemisphere.
Which of the above statements are correct?
a) 1 and 2
b) 2 and 3
c) 1 and 3
d) All of the above
Q.70) Solution (d)
Self explanatory.
A reference from current affairs
http://timesofindia.indiatimes.com/city/goa/A-mid-summer-tryst-with-thestars/articleshow/52368133.cms
Q.71) In cricket, the bowlers are now mastering the art of reverse swinging the ball. This
helps them to take the pitch out of the equation, and move the ball in the air. Which of
the following statements are correct about reverse swing?
1. For reverse swing to happen the ball needs to be rough on one side and shiny on the
other.
2. The ball moves towards the shining side.
3. Due to friction, the air pressure on the rough side is less and shiny side is more.
Select the code from the following:
a) 1 and 2
b) 2 and 3
c) 1 and 3
d) All of the above
Q.71) Solution (a)
The reverse swinging phenomenon can be explained with the help of Bernoullis Principle.
The bowlers deliberately make one side a little rough and polish the other side. This makes
the two sides contrasting. Now when the ball is released, the rough side encounters high
aerodynamic friction, which increases the air pressure on the rough side. On the other hand,
the smooth side reduces the aerodynamic drag, hence less air pressure.
What Bernoulli's principle states is this: (in very very simple words)
The pressure difference will create a net force which has a direction. Remember Pressure is
simply force per area. This force created will make the ball move sideways. Also the ball will
move towards the shiny side, which has less air pressure.

www.iasbaba.com

40

General Science and Technology- Subject wise compilation of 60 Day Plan- 2016

Q.72) Consider the following situation:


If an object is released from a moving train, the object will go:
a) In the opposite direction of the moving train.
b) In the same direction of the moving train.
c) It will fall straight downwards.
d) None of the above
Q.72) Solution (b)
The object will be in inertia of motion. As it was moving with train, it had a component of
velocity in the direction of motion. If it is released, that component will still be there and it
will fall in the direction of motion of the train.
Q.73) Steam burns are more severe than the burns caused by boiling water because:
a) The temperature of steam is higher than the boiling water.
b) Steam contains latent heat.
c) Steam hits the body with a pressure while water does not.
d) The given statement is false.
Q.73) Solution (b)
Steam burns are more severe as steam contains latent heat.
Q.74) Which of the following statements about Amber are correct?
1. It is a fossilized tree resin.
2. It is helpful in paleontology as it contains preserved specimens of plants and insects.
3. It is highly sought after because of its vibrant colour and is used in jewelry.
4. Amber is used as an ingredient in perfumes and healing agent in folk medicines.
Select the code from the following:
a) 1 and 2
b) 1 and 3
c) 2,3 and 4
d) All of the above
Q.74) Solution (d)
Self explanatory. Term is taken from NCERT.
You can also refer to these links
http://timesofindia.indiatimes.com/environment/30-million-year-old-fossil-flowers-foundpreserved-in-amber/articleshow/51026034.cms
http://www.thehindu.com/opinion/op-ed/huge-amber-deposit-discovery-in-westernindia/article850900.ece

www.iasbaba.com

41

General Science and Technology- Subject wise compilation of 60 Day Plan- 2016

Q.75) When cool air flows from a high mountain region to a region of lower elevation, the
air will
a) increase in moisture content
b) condense, forming large amounts of dew
c) undergo adiabatic warming
d) undergo adiabatic cooling
Q.75) Solution (c)
Explanation:
When cool air flows from a high mountain region to a region of lower elevation, the air
undergoes adiabatic warming.
Adiabatic warming occurs as the pressure of the air is increased as it descends.
Q.76) Match List I with List II and select the correct answer using the code given below the
Lists:
List I
List II
A. Smelting

1. Process in which ore is heated usually in


the presence of air, at temperatures below
its melting points

B. Roasting

2. Reduction of oxide ore with carbon at high


temperature

C. Calcination

3. Process in which ore is heated, generally in


the absence of air, to expel water from
hydrated oxide or carbon dioxide from a
carbonate at temperature below their
melting point

D. Corrosion

4. Process of slow conversion of metals into


their undesirable compounds by reaction
with moisture and other gases present in the
atmosphere

A-B-C-D

www.iasbaba.com

42

General Science and Technology- Subject wise compilation of 60 Day Plan- 2016
a)
b)
c)
d)

1-4-3-2
1-4-2-3
2-1-4-3
2-1-3-4

Q.76) Solution (d)


Q.77) IASbabaji is hiking up Mt. Everest, and recently reached the base camp at 12,000
feet. While there, babaji decides to make some tea, and puts a thermometer in the boiling
water. To his surprise, the thermometer tells him the water is boiling at 96 C. Why is this
happening?
a) Babaji was surprised to watch water boiling more quickly.
b) The pressure is lower at higher elevations, and results in a lower boiling temperature.
c) The number of solutes in the air is higher at higher elevations, resulting in boiling point
depression.
d) Babaji is suffering from oxygen deprivation, and is reading the thermometer incorrectly.
Q.77) Solution is (b)
Explanation:
At higher elevations, the atmospheric pressure is lower. At 12,000 feet, the atmospheric
pressure is only 0.70 atm, compared to 1 atm at sea level. The reduction in pressure
leads to a reduction in boiling temperature.
Q.78) Polarized sunglasses are used to cut glare from sunlight reflected at a glancing angle
off cars, water, and other surfaces. Such sunglasses are a practical application of which of
the following physical principles?
a) Brewster's law
b) Lenz's law
c) Coulomb's law
d) Snell's law
Q.78) Solution is (a)
Explanation:
According to Brewsters law, reflected light will always be polarized in a horizontal
direction, parallel to the reflecting surface. Polarized sunglasses are constructed to block
this reflected light and to transmit light polarized only in the vertical direction.
Q.79) Consider the following
1. Evaporation of gas
2. Compression of the gas
3. Solubility of Gas

www.iasbaba.com

43

General Science and Technology- Subject wise compilation of 60 Day Plan- 2016
4. Expansion of the gas
5. Diffusion of gas
Which of the above phenomenon (in liquid medium) will increase with the increase in
Temperature?
a) 1, 2, 3, 4 and 5
b) 1, 4 and 5
c) 1, 3, 4 and 5
d) 2, 4 and 5
Q.79) Solution (b)
NCERT- Class 9th, Chapter 1
In understanding the effects of temperature on the solubility of gases, it is first important to
remember that temperature is a measure of the average kinetic energy. As temperature
increases, kinetic energy increases. The greater kinetic energy results in greater molecular
motion of the gas particles. As a result, the gas particles dissolved in the liquid are more
likely to escape to the gas phase and the existing gas particles are less likely to be dissolved.
The converse is true as well. The trend is thus as follows: increased temperatures mean
lesser solubility and decreased temperatures mean higher solubility.
Le Chatelier's principle allows better conceptualization of these trends. First, note that the
process of dissolving gas in liquid is usually exothermic. As such, increasing temperatures
result in stress on the product side (because heat is on the product side). In turn, Le
Chatelier's principle predicts that the system shifts towards the reactant side in order to
alleviate this new stress. Consequently, the equilibrium concentration of the gas particles in
gaseous phase increases, resulting in lowered solubility.
Conversely, decreasing temperatures result in stress on the reactant side (because heat is
on the product side). In turn, Le Chtelier's principle predicts that the system shifts toward
the product side in order to compensate for this new stress. Consequently, the equilibrium
concentration of the gas particles in gaseous phase would decrease, resulting in greater
solubility.
Q.80) Recently scientists have found a way to lock CO2 by turning it into harmless rock.
Which of the following statements are correct about this process?
a) The CO2 is injected into volcanic bedrock where it will react to form carbonate rocks.
b) CO2 is captured and solidified. This solid CO2 is then compressed to give it permanent
form.
c) CO2 is reacted with amorphous calcium to form calcium carbonate.
d) Carbon di oxide is captured and stored in Ocean trench where it gets solidified due to
high pressure.

www.iasbaba.com

44

General Science and Technology- Subject wise compilation of 60 Day Plan- 2016

Q.80) Solution (a)


The greenhouse gas carbon dioxide (CO2) can be permanently and rapidly locked away from
the atmosphere, by injecting it into volcanic bedrock. The CO2 reacts with the surrounding
rock, forming environmentally benign minerals.
Geoengineers have long explored the possibility of sealing CO2 gas in voids underground,
such as in abandoned oil and gas reservoirs, but these are susceptible to leakage. So
attention has now turned to the mineralisation of carbon to permanently dispose of CO2.
Q.81) Touch screens now have become an important component of many electronic
devices like mobiles, ATM machines, laptops etc. Consider the following statements
regarding touch screens:
1. Commonly there are two types of touch screens- resistive and capacitive.
2. Capacitive touch screens are multilayered with inner most and outer most layers
behaving as a conductor.
3. Resistive screens can detect more than one touch at once at different spots.
Which of the above statements are correct?
a) 1 and 2
b) 2 and 3
c) 1 and 3
d) All of the above
Q.81) Solution (a)
There are several types of touch screen technologies available, but of these, two are most
common. The first is the Resistive touch screen which works a bit like transparent
keyboard overlaid on top of the screen. There's a flexible upper layer of conducting
polyester plastic bonded to a rigid lower layer of conducting sheet and these are separated
by an insulating membrane. When the screen is pressed at a specific spot, it forces the
polyester to touch the glass and complete a circuit just like pressing the key on a
keyboard. A chip inside the screen figures out the coordinates of the place which is touched.
The other is the capacitive type screen. These screens are made from multiple layers. The
inner layer conducts electricity and so does the outer layer. Effectively the screen behaves
like two electrical conductors separated by an insulator in other words, a capacitor.
When the user brings his finger up to the screen, it alters the electrical field by a certain
amount that varies according to where the finger is. Capacitive screens can be touched in
more than one place at once.

www.iasbaba.com

45

General Science and Technology- Subject wise compilation of 60 Day Plan- 2016
Q.82) Recently Zika virus caused a havoc in South American and central American
countries. Few cases have also been registered in USA. Which of the following statements
are correct about Zika virus?
1. It is disease caused by virus from infected Aedes mosquito.
2. Zika virus can be sexually transmitted.
3. Virus can pass through the fetus of a pregnant woman and can infect the brain cells of
the fetus.
4. The symptoms are similar to other arbovirus infections such as dengue, and include
fever, skin rashes, conjunctivitis, muscle and joint pain, malaise, and headache.
Select the code from the following:
a) 1,3 and 4
b) 1 and 3
c) 3 and 4
d) All of the above
Q.82) Solution (d)
Zika virus disease is caused by a virus transmitted primarily by Aedes mosquitoes.
People with Zika virus disease can have symptoms including mild fever, skin rash,
conjunctivitis, muscle and joint pain, malaise or headache. These symptoms normally last
for 2-7 days.
Sexual transmission of Zika virus has been documented in several different countries. To
reduce the risk of sexual transmission and potential pregnancy complications related to Zika
virus infection, the sexual partners of pregnant women, living in or returning from areas
where local transmission of Zika virus occurs should practice safer sex (including using
condoms) or abstain from sexual activity throughout the pregnancy.
Q.83) A two foot long but discovered in China has been declared as the worlds longest
insect. Which of the following species does this bug belongs to?
a) A Mantis
b) A Stick Insect
c) A centipede
d) A titan beetle

www.iasbaba.com

46

General Science and Technology- Subject wise compilation of 60 Day Plan- 2016
Q.83) Solution (b)
A bug, measuring over half-a-metre long, discovered in southern China has been declared
the world's longest insect.
A stick insect measuring 62.4 centimetres found two years ago in the southern province of
Guangxi has broken the record for length amongst the world's 807,625 known insects, the
official Xinhua agency said, citing the Insect Museum of West China.
Q.84) Consider the following statements:
1. A black hole is a place in space where gravity pulls so much that even light cannot get
out.
2. Density of black hole is very high.
3. Black holes are made when the fuel of a star gets over and it becomes a white dwarf.
Which of the above statements are correct?
a) 1 and 2
b) 2 and 3
c) 1 and 3
d) All of the above
Q.84) Solution (a)
A black hole is a place in space where gravity pulls so much that even light cannot get out.
The gravity is so strong because matter has been squeezed into a tiny space. This can
happen when a star is dying.
Because no light can get out, people can't see black holes. They are invisible. Space
telescopes with special tools can help find black holes. The special tools can see how stars
that are very close to black holes act differently than other stars.
Stellar black holes are made when the center of a very big star falls in upon itself, or
collapses. When this happens, it causes a supernova. A supernova is an exploding star that
blasts part of the star into space. Scientists think supermassive black holes were made at the
same time as the galaxy they are in.
Small stars become white dwarfs when there fuel gets over.
Q.85) Hepatitis is the medical term for inflammation of the liver. Consider the following
regarding this
1. There are five types of viruses responsible for Hepatitis
2. Intake of toxic substances, alcohol and certain drugs are some of the causes of Hepatitis
3. Hepatitis B is transmitted through exposure to infective blood, semen, and other body
fluids.
Select the correct code
a) 1 and 2

www.iasbaba.com

47

General Science and Technology- Subject wise compilation of 60 Day Plan- 2016
b) 2 and 3
c) 1 and 3
d) 1, 2 and 3
Q.85) Solution (d)
http://www.thehindu.com/news/cities/bangalore/hepatitis-b-virus-more-prevalent-thanhiv-in-donor-blood/article8725933.ece
http://www.thehindu.com/news/cities/chennai/its-a-long-road-to-recovery-for-hepatitis-cpatients-at-stanley/article8553352.ece
http://www.who.int/features/qa/76/en/
http://www.thehindu.com/sci-tech/health/two-million-people-coinfected-with-hivhepatitis-c-globally/article8341900.ece
Q.86) Consider the following statements with respect to Maglev Technology
1. Maglev trains does not have engine.
2. It does not emit greenhouse gases.
3. It is generally preferred for short distance commutation.
Which of the above given statements is/are correct?
a) 2 only
b) 1 & 2 only
c) 2 & 3 only
d) None of the above
Q.86) Solution (b)
Maglev Technology is generally used for Mono rails which doesnt operates based upon
engines. Moreover the Magnetic levitation doesnt use any kind of fossil fuels so it wont
release any of the greenhouse gases.
Monorail system is preferred for long distance commutation only metro rail is preferred
for short distance commutation.
Q.87) Consider the following statements about Bio Digester Technology
1. Its a green technology developed by CSIR(Council of Scientific & Industrial Research)
2. It uses anaerobic digestion method.
3. Process is carried out by using Bacteria and Fungi.
Which of the above given statements is/are correct?
a) 2 only
b) 1 & 2 only
c) 2 & 3 only
d) 1,2 & 3

www.iasbaba.com

48

General Science and Technology- Subject wise compilation of 60 Day Plan- 2016

www.iasbaba.com

49

General Science and Technology- Subject wise compilation of 60 Day Plan- 2016
Q.87) Solution (a)
Bio-digester technology is been carried out only with the help of bacteria and not by
using fungi. Moreover the same question can be asked in the perspective of Dark
Fermentation.
It is developed by DRDO
Q.88) Consider the following statements with respect to Green Diesel
1. Green Diesel is generally referred to as Biodiesel.
2. It is produced through Hydro cracking technology.
3. It possesses same chemical properties as petroleum based diesel.
Which of the above given statements is/are not correct?
a) 1 only
b) 2 only
c) 1 & 2 only
d) 1, 2 & 3
Q.88) Solution (a)
Green diesel is entirely different from the Bio diesel.
The major difference between them is the process of making. Biodiesel is processed by
using transesterification, while the green diesel is processed by fractional distillation like
fossil origin by Hydrocracking technology.
Moreover the chemical composition between them is also different.
Green diesel possesses same chemical properties as petroleum based diesel.
Q.89) Consider the following w.r.t Semiconductors
1. Semiconducting materials are always crystalline solids
2. Semiconductors are always manmade and properties are tweaked by Doping
3. A semiconductor is a material whose electrical conductivity decreases as the
temperature increases
Select the incorrect code
a) 1 and 2 only
b) 2 and 3 only
c) 1 and 3 Only
d) All
Q.89) Solution (d)
Hint for this question- http://www.thehindu.com/todays-paper/tp-business/globalsemiconductor-capital-spending-to-decline-2-in-2016/article8578400.ece

www.iasbaba.com

50

General Science and Technology- Subject wise compilation of 60 Day Plan- 2016
Semiconductors are crystalline or amorphous solids with distinct electrical characteristics.
Semiconductors are substances that only conduct electricity under certain conditions and
include Silicon, Germanium, and sometimes tin. They are semiconductors that occur
naturally and do not require any sort of chemical doping and can often be recognized by
characteristic crystal lattice structures that they form.
A semiconductor is a material whose electrical conductivity increases with increasing
temperature. This broad definition distinguishes semiconductors from metals, whose
electrical conductivity decreases as the temperature increases. In general, there are two
basic classifications of semiconductors: intrinsic semiconductors and extrinsic
semiconductors. Unlike extrinsic semiconductors, intrinsic semiconductors are naturally
occurring elements within nature. The defining characteristic of these intrinsic
semiconductor elements is their four valence electrons each occupying a different orbital.
Q.90) TERAHERTZ imaging has recently attracted attention for its ability to see through
everyday objects. It exists between
a) Microwave and Radio waves wavelengths
b) Radio waves and Infrared wavelengths
c) Microwaves and Infrared wavelengths
d) Infrared and Visible wavelengths
Q.90) Solution (c)
TERAHERTZ imaging, which exists between the microwave and infrared wavelengths, has
recently attracted attention for its ability to see through everyday objects. Among other
things, it has been used to detect defects in space shuttle panels and uncover the material
composition and substructure of paintings and murals. Current terahertz devices remain
inherently slow, invasive, and better suited to non-biological materials. Besides, they are
extremely expensive to manufacture.
Hinthttp://www.frontline.in/science-and-technology/advances-in-terahertzimaging/article8745879.ece
Q.91) There is a decline in the population of pollinators as per recent research and
findings. What could be the possible reasons for this?
1. Use of pathogens
2. Use of street lights
3. Air pollution
4. Misuse of pesticides
5. Change in seasonal behaviour due to global warming
Select the appropriate code
a) 1, 2, 4 and 5

www.iasbaba.com

51

General Science and Technology- Subject wise compilation of 60 Day Plan- 2016
b) 1, 3, 4 and 5
c) 1, 3 and 4
d) 1, 2, 3, 4 and 5
Q.91) Solution (d)
Hinthttp://www.frontline.in/science-and-technology/moths-streetlighting/article8745865.ece
https://en.wikipedia.org/wiki/Pollinator_decline#Consequences
Q.92) Project Baseline is associated with which of the following?
a) Seeds
b) Invasive Species
c) Drugs
d) Microchips
Q.92) Solution (a)
Hintcapsule/article8466314.ece

http://www.frontline.in/science-and-technology/seed-time-

Q.93) 3D Printing has gained lot of attention in recent times. Consider the following
statements regarding 3D Printing
1. 3D printing is a process of making three dimensional solid objects from anything
2. It is also known as Additive Manufacturing (AM)
3. 3D printing can create in a wide range of materials that include thermoplastics,
thermoplastic composites, pure metals, metal alloys, ceramics and various forms of
food.
Select the incorrect code
a) Only 1
b) Only 2
c) 2 and 3
d) 1 and 2
Q.93) Solution (a)
3D printing is a process of making three dimensional solid objects from a digital file not
ANYTHING
Hint
http://www.thehindu.com/news/international/3d-printing-could-help-fix-damagedcartilage-in-knees/article8365269.ece
http://www.thehindu.com/news/cities/Visakhapatnam/3d-printing-to-make-healthcareaffordable/article8231694.ece

www.iasbaba.com

52

General Science and Technology- Subject wise compilation of 60 Day Plan- 2016

Q.94) Pick the odd one out in regard to its stem type:
a) Bamboo
b) Sugar cane
c) Banana
d) Mango
Q.94) Solution (d)
Explanation:
Different types of stems
(a) Monocots: Secure stems of several
plants such as bamboo, sugar cane,
banana and corn. If we cut each of the
stems crosswise with a very sharp knife
or razor blade, we can observe the
similarities in the cut cross sections.
Especially we can notice that the tubes
or fibro-vascular bundles are scattered
throughout the pith on the inside of the
stem.
(b) Dicers/Dicots: Secure the stems of
several plants or small trees such as
willow, tomato, mango etc. Cut across
each of these stems with a sharp knife
or razor blade, we can observe that just
under the outside layer of the stem
there is a bright green layer. This is the
cambium layer. Also we can observe
that the tubes or fibro- vascular bundles are arranged in a ring about the central, or woody
portion of the stem.
Doubts: banana is a monocot plant and not dicot, bambo, sugarcane, corn n banana all are
monocots and vascular bundles in all the monocots are scattered
Clarification: There was a factual error. Corrections made to the questions. Please update.

Q.95) Match List I with List II and select the correct answer using the code given below the
Lists:
List I List II
A. Magnetic Flux 1. Tesla
B. Magnetic Induction 2. Candela
C. Luminous intensity 3. Coulomb

www.iasbaba.com

53

General Science and Technology- Subject wise compilation of 60 Day Plan- 2016
D. Electric Charge 4. Weber
E. Inductance 5. Henry
A-B-C-D-E
a) 4-3-5-2-1
b) 4-1-2-3-5
c) 1-4-2-3-5
d) 1-4-2-5-3
Q.95) Solution (b)
Explanation:
A. Magnetic Flux
B. Magnetic Induction
C. Luminous intensity
D. Electric Charge
E. Inductance

1. Weber
2. Tesla
3. Candela
4. Coulomb
5. Henry

Q.96) Haemophilic man marries a normal woman. Their offsprings will be


a) all girls haemophilic
b) all normal
c) all haemophilic
d) all boys haemophilic
Q.96) Solution (b)
Explanation:

Daughters and sons both will be phenotypically normal.


Q.97) Consider the following in regard to Dolphins and Porpoise:
1) Both are cetaceans and closely related to whales, but both belong to different families.
2) Both are highly intelligent, they have large complex brains and are self-aware like
humans
3) Dolphins tend to have prominent, elongated beaks and cone-shaped teeth, while
porpoises have smaller mouths and spade-shaped teeth.

www.iasbaba.com

54

General Science and Technology- Subject wise compilation of 60 Day Plan- 2016
4) Dolphins have hooked or curved dorsal fin (the one in the middle of the animal's back),
while the porpoises have triangular dorsal fin.
Which of the statements given above is/are correct?
a) 3 and 4 only
b) 1, 2 and 3 only
c) 1 and 2 only
d) All of the above
Q.97) Solution (d)
Explanation:
Factual question All given statements are correct and self-explanatory

Q.98) The use of nanobots in healthcare, a fictional theory, is a reality now. Consider the
following statements:
1. Nanobots are tiny robots that can be directly injected in the blood stream and act like
white blood cells and destroy bacteria and other pathogen.
2. Nanobots will have their own sensors and propulsion system.
3. They can be used to deliver targeted medicine with minimum side effect.

www.iasbaba.com

55

General Science and Technology- Subject wise compilation of 60 Day Plan- 2016
Select the code from the following:
a) 1 only
b) 1 and 2
c) 2 and 3
d) All of the above
Q.98) Solution (d)
Although nanobots are far from being utilized today, but the future is coming where these
tiny robots can function like our own white blood cells and destroy bacteria and other
pathogens.
These miniature robots would function like
their full-size equivalents with their own
sensors, and propulsion systems and could
perform small tasks like delivering
chemotherapy 1000 times more powerful
than using drugs and would not cause as
many side-effects to patients like the
current treatments do.
Other specific types of nanobots that are
being developed are Microbivore,
Respirocyte, Clottocyte, and Cellular repair
nanobots that can destroy bacteria, carry
oxygen, create blood clots for wounds,
and repair cells.
Q.99) C. V. Raman, was the first Indian Scientist to receive a Nobel Price. He published his
theory on the Raman Effect in 1928. Raman Effect is associated with which of the
following?
a) Scattering of light
b) Total internal reflection
c) Atomic structure
d) Semi-conductors
Q.99) Solution (a)
Raman Effect, change in the wavelength of light that occurs when a light beam is deflected
by molecules.
When a beam of light traverses a dust-free, transparent sample of a chemical compound, a
small fraction of the light emerges in directions other than that of the incident (incoming)
beam. Most of this scattered light is of unchanged wavelength. A small part, however, has

www.iasbaba.com

56

General Science and Technology- Subject wise compilation of 60 Day Plan- 2016
wavelengths different from that of the incident light; its presence is a result of the Raman
Effect.
Q.100) Consider the following statements regarding microwave ovens:
1. Instead of generating heat that warms the food from outside, the microwaves penetrate
food and create the heat within.
2. Microwaves are electromagnetic waves that are created by a component called
Megatron in the oven. Megatron has vacuum tubes which creates the microwave.
3. Microwaves are known to destroy the nutrients in the food.
Which of the above statements are correct?
a) 1 and 2
b) 2 and 3
c) 1 and 3
d) All of the above
Q.100) Solution (a)
The key component of a microwave oven is the magnetron. Although the name conjures up
hardware from a questionable science-fiction movie, the sophisticated vacuum tube
generates microwaves powerful enough for military radars (for which it was originally
developed). Instead of a flame or electric coil generating heat that warms food from the
outside, the microwaves penetrate food and create heat from within.
Q.101) Early Career Research Award scheme aims to provide quick research support to
the young researchers who are in their early career for pursuing exciting and innovative
research in frontier areas of science and engineering. Consider the following statements
regarding this scheme:
1. The research grant is 50 Lakhs, including overhead charges for travelling, manpower,
research equipment etc.
2. The award is given in three installments for three years.
3. The award is only for Indian citizens.
Which of the above statements regarding the scheme are incorrect?
a) 1 and 2
b) 3 only
c) 1 and 3
d) None of the above
Q.101) Solution (a)

www.iasbaba.com

57

General Science and Technology- Subject wise compilation of 60 Day Plan- 2016

The Early Career Research for the purpose of the program refers to the first assignment
of the applicant in a regular capacity in a recognized academic institution or national
laboratory or any other recognized R & D institution in India.
The Early Career Research Award is a one-time award and carries a research grant up to
Rs. 50 Lakhs (excluding overheads) for a period of three years.
The research grant covers equipment, manpower, consumables, manpower, travel and
contingency apart from overheads.

Eligibility :
The applicant should be an Indian citizen.
The applicant should hold Ph.D. degree in Science or Engineering or M.D or M.S degree
in any area of medicine.
The scheme encourages young researchers at the start of their independent research
careers. The applicant, therefore, must hold a regular academic/research position in a
recognized academic institution/ or national laboratories or any other recognized R&D
institutions and must apply not later than the first two years of their regular service.
Q.102) Japanese Encephalitis is a notifiable disease in India. Which of the following
statements are correct about JE?
1. Japanese Encephalitis is a caused by flavivirus transmitted through infected Culex
mosquitoes.
2. JE virus is neurotorpic and arbovirus and primarily affects central nervous system.
3. India has launched JENVAC, its first indigenous vaccine to protect children from JEV.
4. JANVAC has been induced in National Immunisation Program.
Which of the above statements are correct?
a) 1 and 2
b) 1,2 and 3
c) 2,3 and 4
d) All of the above
Q.102) Solution (d)
Factual question. Self Expalnatory.
Note: Read about National Immunisation Program and diseases covered in it. A direct
question can be asked.
Q.103) Which of the following forms of Carbon, is a good conductor of electricity?
1. Diamond
2. Graphite
3. Graphene
4. Charcoal

www.iasbaba.com

58

General Science and Technology- Subject wise compilation of 60 Day Plan- 2016

Select the code from the following:


a) 2 and 3
b) 1 and 2
c) 3 and 4
d) 2 only
Q.103) Solution (a)
Graphene is a 2D nanostructure of carbon which is capable of conducting electricity and
offers very less resistance. Research is being going on to use graphene in solar panels and
touch screens to increase efficiency.
Q.104) Rustom II developed by DRDO is an important aeronautical achievement of India.
Which of the following statements regarding Rustom II are correct?
1. It is remotely piloted UAV in the category of Medium Altitude, Long Endurance (MALE)
category.
2. Rustom II is a fully featured, Combat capable drone built on the lines of US predator
drones.
Select the code from the following:
a) 1 only
b) 2 only
c) Both 1 and 2
d) Neither 1 nor 2
Q.104) Solution (c)
Both the options are correct regarding Rustom II.
http://www.militaryfactory.com/aircraft/detail.asp?aircraft_id=985
Q.105) Twinkle Twinkle little star. Consider the following statements regarding
twinkling of stars:
1. The scientific name for the twinkling of stars is stellar scintillation.
2. It is because of hot and cold cycles of the stars where it shines bright and low
alternately. Although it takes millions of years for the cycles to complete on stars, it
takes fraction of a second on earth to appear.
Which of the above statement(s) is/are correct?
a) 1 only
b) 2 only
c) Both 1 and 2
d) Neither 1 nor 2

www.iasbaba.com

59

General Science and Technology- Subject wise compilation of 60 Day Plan- 2016

Q.105) Solution (a)


The scientific name for the twinkling of stars is stellar scintillation (or astronomical
scintillation). Stars twinkle when we see them from the Earth's surface because we are
viewing them through thick layers of turbulent (moving) air in the Earth's atmosphere. Most
scintillation effects are caused by anomalous refraction caused by small-scale fluctuations in
air density usually related to temperature gradients.
Q.106) Which of the following statements are correct about the phenomenon of Total
Internal Reflection (TIR)?
1. It is more efficient than normal reflection as light energy is not wasted by absorption at
the interface.
2. Optical fibers are based on the principle of Total Internal Reflection.
3. The depth of the bottom under water appears relatively shallow than it actually is,
because of TIR.
Select the code from below:
a) 1 and 2
b) 2 and 3
c) 1 and 3
d) All of the above
Q.106) Solution (a)
When light goes from a denser medium to a less dense medium, as the angle of incidence
exceeds the critical angle, the ray reflects back to the denser medium. This phenomenon is
called Total Internal Reflection.
Total Internal Reflection is a very efficient reflection, as the loss of light energy is almost
negligible.
Q.107) Tywin Lanister wanted to reshape the Valyrian steel sword of Eddard Stark. He
started heating the sword in the furnace. As it was heated, the sword started glowing in
different colours. At what colour do you think the temperature of the sword was
maximum?
a) Red
b) Blue
c) Orange
d) Yellow
Q.107) Solution (b)

www.iasbaba.com

60

General Science and Technology- Subject wise compilation of 60 Day Plan- 2016
As the temperature of a metal is increased, it starts emitting radiations in visible spectrum.
Higher the temperature, shorter will be the wavelength. Since the wavelength of blue is
shortest in the given colours, the temperature of metal was highest at blue.
Q.108) Consider the following statements regarding Mercury:
1. It is only metal which stays in liquid state at standard conditions of Temperature and
Pressure.
2. It is commonly known as Quicksilver
3. It is used in thermometers and Barometers as it is a good conductor of Heat.
4. It is used in fluorescent lighting lamps.
Which of the above statements are correct?
a) 1,3 and 4
b) 1,2 and 4
c) 2,3 and 4
d) All of the above
Q.108) Solution (b)
Mercury is a bad conductor of heat. It is used in different transparent meters because
It has more cohesion than adhesion i.e. It does not stick to the walls of the tube.
It is shiny in colour and bulging out meniscus, so easier to look from outside the flask
and give correct reading.
It expands with small change of temperature.
Q.109) Indian Navy has successfully inducted Varunastra a heavy weight anti-submarine
Torpedo. Which of the following statements are correct about Varunastra?
1. It has been jointly developed by India and Israel.
2. It is capable of targeting stealthy and quiet submarines, both in deep and littoral waters
in intense counter-measure environment.
3. It has a GPS-based locating aid.
4. it is capable of carrying Nuclear Warhead
Select the code from the following:
a) 1,2 and 3
b) 2 and 3
c) 2,3 and 4
d) All of the above
Q.109) Solution (b)
The Varunastra torpedo has been developed by Naval Science and Technological
Laboratory (NSTL), a premier laboratory of DRDO.

www.iasbaba.com

61

General Science and Technology- Subject wise compilation of 60 Day Plan- 2016

It is not capable of carrying nuclear warhead.


It is capable of targeting stealthy and quiet submarines, both in deep and littoral waters
in intense counter-measure environment.
It has advanced autonomous guidance algorithms with low drift navigational aids,
insensitive warhead which can operate in various combat scenarios.
It has integrated instrumentation system for recording its all the dynamic parameters in
case of emergency shut down or malfunction.
It has a GPS-based locating aid which is a unique feature in contemporary torpedoes in
the world.
The torpedo can be launched from Delhi, Kolkata, Teg, Talwar and Kamorta classes of
warships.

Q.110) Consider the following statements:


1. When we breathe in, our diaphragm contracts.
2. When we focus at a far object, our pupils dilate.
Which of the above statement(s) is/are incorrect?
a) 1 only
b) 2 only
c) Both 1 and 2
d) Neither 1 nor 2
Q.110) Solution (d)
Both the statements are correct.
When you breathe in, or inhale, your diaphragm contracts (tightens) and moves
downward. This increases the space in your chest cavity, into which your lungs expand.
Rays of light from a far away object are almost perfectly parallel and by the time they
reach your eye, these rays are quite weak, so the pupils dilate to let as much light in
from the object as possible.
Q.111) The rain bearing thick clouds are black/grey. What reason can be attributed to
this?
a) They have a lot of dust particles and other impurities which gives them dark colour.
b) Sunlight is unable to penetrate these clouds. It is either scattered or absorbed. Hence
they appear dark to an observer on earth.
c) Lightning burns the cloud to give them dark colour.
d) A cloud absorb large quantities of NOx and SOx which are acidic in nature and gives the
cloud their dark colour.
Q.111) Solution (b)

www.iasbaba.com

62

General Science and Technology- Subject wise compilation of 60 Day Plan- 2016
Most clouds appear white because sunlight reflects off the water droplets. If the clouds are
thick, the droplets scatter or absorb the light and less solar radiation can travel through
them. This is why storm clouds are dark black or gray.
Q.112) In a bollywood movie, hero was chased by a villain. Villain shot him. The bullet hit
the hero and got stuck in the body. After looking at the wound, the doctor said because
the bullet got stuck in the body, it has spread the poison.
What does this spreading of poison by bullet mean?
a) The bullet tips are poisoned to kill the target. When this poison touches the body it is
spread quickly.
b) When the bullet is stuck in the body, the high kinetic energy of bullet gets converted
into heat energy. Because of the generation of this immense heat, the surrounding area
gets burnt.
c) The bullets are usually made of lead. When they get stuck in the body they cause lead
poisoning.
d) Bullets have a tendency to react with blood. When they get stuck, they get more time to
react and cause poisoning in the blood stream.
Q.113) Solution (c)
Lead poisoning is an unusual complication of retained bullet fragments. The bullets and
shotgun pellets most frequently encountered are 50100 percent lead with an outer
material made of a metal or alloy harder than lead (copper, nickel, steel, or aluminum)
Q.114) Light pollution, also known as photopollution, has emerged as a new menace for
the environment. Which of the following statements are correct about it?
1. Excessive brightening of the night sky by street lights and other man made sources is
called light pollution.
2. Artificial lights overpower the darkness and affect the natural day-night pattern putting
pressure on biological process of plants and animals.
3. It obstructs the vision of night sky.
Select the code from the following:
a) 1 only
b) 1 and 3
c) 2 and 3
d) All of the above
Q.114) Solution (d)
The inappropriate or excessive use of artificial light known as light pollution can have
serious environmental consequences for humans, wildlife, and our climate. Components of
light pollution include:

www.iasbaba.com

63

General Science and Technology- Subject wise compilation of 60 Day Plan- 2016

Glare excessive brightness that causes visual discomfort


Skyglow brightening of the night sky over inhabited areas
Light trespass light falling where it is not intended or needed
Clutter bright, confusing and excessive groupings of light sources

Q.115) Consider the following statements regarding JUNO mission of NASA:


1. JUNO stands for Jupitor Near Polar Orbitor.
2. It is the first mission to Jupitor by NASA.
3. The orbitor will study Jupitors composition, gravity field, magnetic field, and polar
magnetosphere.
Which of the above statements are correct?
a) 1 and 3
b) 1 and 2
c) 2 and 3
d) All of the above
Q.115) Solution (a)
The Jupiter Near-polar Orbiter (Juno) is a NASA New Frontiers mission currently en route to
the planet Jupiter. Juno was launched from Cape Canaveral Air Force Station on August 5,
2011 and will arrive on July 4, 2016. The spacecraft is to be placed in a polar orbit to study
Jupiter's composition, gravity field, magnetic field, and polar magnetosphere. Juno will also
search for clues about how the planet formed, including whether it has a rocky core, the
amount of water present within the deep atmosphere, how its mass is distributed, and its
deep winds, which can reach speeds of 618 kilometers per hour (384 mph).
Juno will be the second spacecraft to orbit Jupiter, following the Galileo probe which orbited
from 19952003.
Q.116) The National Green Tribunal issued notices to Union Health, environment and
water resource ministries seeking ban on the use of microplastics. Which of the following
statements regarding microplastics are correct?
1. It is used to describe plastic particles with the upper size limit of 5 micro meters.
2. It is widely used in cosmetics as exfoliating agents and in personal care products such as
toothpastes, facewash etc.
3. They are small enough to flow through sewer systems and end up in seas and oceans.
4. They are highly non-biodegradable.
Select the code from the following:
a) 1,2 and 3
b) 2,3 and 4

www.iasbaba.com

64

General Science and Technology- Subject wise compilation of 60 Day Plan- 2016
c) 1,3 and 4
d) All of the above
Q.116) Solution (b)
Microplastics is the term used to describe plastic particles with maximum size of 5mm.
Q.117) Nano-architecture is a breakthrough in the field of material science. Consider the
following statements regarding Nano-Architecture:
1. It is creation of precise city plans and building models at nano-scale.
2. These are the Materials whose structures can be precisely tailored so they are strong yet
flexible and extremely light.
3. These are ceramics which are non brittle and if produced on large scale can replace
composites.
Which of the above statements are correct?
a) 1 only
b) 2 and 3
c) 1 and 3
d) All of the above
Q.117) Solution (b)
Nano-Architecture
Breakthrough Materials whose structures can be precisely tailored so they are strong yet
flexible and extremely light.
Why It Matters Lighter structural materials would be more energy-efficient and
versatile.
Q.118) Which of the following is the correct order for air passing through the lungs?
a) trachea, alveoli, bronchi, bronchioles
b) bronchus, alveoli, trachea, bronchioles
c) trachea, bronchus, bronchiole, alveoli
d) bronchus, trachea, bronchioles, alveoli
Q.118) Solution (c)
Explanation:
Observe the figure and identify the correct order of air passing through the lungs

www.iasbaba.com

65

General Science and Technology- Subject wise compilation of 60 Day Plan- 2016

Q.119) Light waves are composed of both electric and magnetic field is proposed by
a) Newtons corpuscular theory
b) Huygen's wave theory
c) Maxwell's theory of light
d) Planks theory of light
Q.119) Solution (c)
Explanation:
Maxwells most significant scientific achievement was his electromagnetic theory of light
propagation which he first presented in 1864 with the publication of A Dynamical
Theory of
the Electromagnetic Field. This paper hypothesised that an electric field, a magnetic
field and light could all be explained with the using a single theory.
Maxwell understood the significance of Faraday's work and realised that the speed of an
electromagnetic waves travelled at the speed of light. As a result, he was able to
incorporate light, magnetism and electricity into a single theory.

www.iasbaba.com

66

General Science and Technology- Subject wise compilation of 60 Day Plan- 2016

Maxwell further concluded that light propagated in electric and magnetic waves, which
he believed would vibrate perpendicular to one another.

Maxwell's electromagnetic theory of light propagation eventually paved the way for a
number of major technological innovations.
The first and possibly most significant of these occurred in 1888, when Heinrich Hertz
used Maxwell's theory to create instruments capable of sending and receiving radio
pulses.
This discovery, contributed to the creation of the television and the microwave and
without Maxwell's tireless efforts, many of the modern conveniences upon which
society has come to depend would not exist.

Q.120) Match List-I with List-II and select the correct answer using the code given below
the Lists:
List - I List - II
A) centrifugal force
1) Along the axis of rotation
B) Centripetal force
2) Towards the centre of rotation
C) Tangential force
3) Away from centre of rotation
D) Angular velocity
4) changes the angular velocity
Choose the appropriate code:
A-B-C-D
a) 3-2-4-1
b) 2-3-4-1
c) 2-3-1-4
d) 3-2-1-4
Q.120) Solution (a)
Explanation:
Factual question - Self-explanatory
Q.121) A stone is thrown straight upward and at the top of its trajectory its velocity is
momentarily zero.
What is its acceleration at this point?
a) Unable to determine
b) Zero
c) 9.8 m/s2 down
d) 9.8 m/s2 up
Q.121) Solution (c)
Explanation:

www.iasbaba.com

67

General Science and Technology- Subject wise compilation of 60 Day Plan- 2016

Basic Concepts: The gravitational acceleration near the surface of the earth is considered
constant, for all practical purposes. This acceleration of 9.8 m/s2 is pointing downward.
Solution: To illustrate how it works, lets take, for example, an upward initial velocity of
9.8 m/s. One second later the velocity will be zero. One second after that it will be at
9.8 m/s. In other words, in each second the velocity is decreased by 9.8 m/s.

Q.122) When would a human body be most likely to respire anaerobically?


a) When watching TV
b) When running a marathon
c) When asleep
d) When eating
Q.122) Solution (b)
Explanation:
Running a marathon will cause the body to demand more oxygen than the lungs can
provide.
Q.123) Match List-I
the Lists:
List I
A) Static friction
B) Limiting friction
C) Kinetic friction
D) Rolling friction

with List-II and select the correct answer using the code given below
List II
1) Constant for a given pair of surfaces
2) lndependent of area of contact
3) SeIf adjusting
4) Has the least magnitude for a given normal reaction

Choose the appropriate code:


A-B-C-D

www.iasbaba.com

68

General Science and Technology- Subject wise compilation of 60 Day Plan- 2016
a)
b)
c)
d)

4-2-4-3
4-1-3-2
2-3-1-4
3-1-2-4

Q.123) Solution (d)


Explanation:
Link: Different Types of Friction Static Friction, Kinetic Friction and Rolling Friction
Q.124) India has set a new record by injecting 20 satellites in one launch using PSLVC34
rocket. Which of the following countries have sent their satellites in this launch apart from
India?
1. US
2. Canada
3. Germany
4. France
5. Indonesia
Select the code from following:
a) 1,2,3 and 4
b) 1,2,3 and 5
c) 2,3,4 and 5
d) All of the above
Q.124) Solution (b)
India took a big leap in space technology on Wednesday when Indian Space Research
Organization (ISRO) used its workhorse PSLV-C34 to inject 20 satellites including 17 foreign
satellites into orbit in a single mission and set a new record on Wednesday.
The 320-tonne Polar Satellite Launch Vehicle (PSLV-C34) took off on its 36th flight at 9.26am
from the Satish Dhawan Space Centre with 20 satellites including its primary payload
Cartosat-2 series, which provides remote sensing services, and earth observation and
imaging satellites from US, Canada, Germany and Indonesia. It was also 14th flight of PSLV in
'XL'configuration with the use of solid strap-on motors.
Q.125) Consider a battleship simultaneously firing two shells (with the same muzzle
velocity) at two enemy ships A and B, as shown in figure below:

www.iasbaba.com

69

General Science and Technology- Subject wise compilation of 60 Day Plan- 2016

If the shells follow the parabolic trajectories shown in the figure, which ship gets hit first?
a) A
b) B
c) both at the same time
d) need more information
Q.125) Solution (b)
Explanation:
The time interval for the entire projectile motion is given by
ttrip = trise + tfall = 2trise
where, trise is the rising time from 0 to the maximum height, and tfall the falling time from
h to 0.
In the absence of air resistance

So the smaller is h, the smaller is ttrip. In other words, B should get hit first.

Q.126) Consider the following statements regarding LIDAR technology:


1. It is the acronym for Laser Image Detection and Ranging.
2. It is a surveying and remote sensing method to measure ranges on earth.
3. The system works on the principle of Radar, but uses light from a Laser.
Which of the above statements are correct?
a) 1 and 2
b) 2 and 3
c) 1 and 3
d) All of the above

www.iasbaba.com

70

General Science and Technology- Subject wise compilation of 60 Day Plan- 2016

Q.126) Solution (b)


LIDAR, which stands for Light Detection and Ranging, is a remote sensing method that uses
light in the form of a pulsed laser to measure ranges (variable distances) to the Earth.
Q.127) Which of the following statements correctly describe the famous Gravitational
Lensing Effect given by Einstein?
1. Light coming from one object will bend due the presence of mass between the object
and the observer.
2. The effect is observed in space where light is bent under the influence of the mass of
galaxies and dark matter.
Select the code from the following:
a) 1 only
b) 2 only
c) Both 1 and 2
d) Neither 1 nor 2
Q.127) Solution (c)
A gravitational lens refers to a distribution of matter (such as a cluster of galaxies) between
a distant source and an observer that is capable of bending the light from the source, as it
travels towards the observer.
Q.128) Car- to- Car communication is a new technology being developed to reduce car
accidents. Consider the following statements regarding Car-to-car communication:
1. Cars will have small radars to judge the speed of vehicles around them and manoeuvre
accordingly.
2. This technique will only work in automatic driving when, driver is not steering the car
and car is on auto pilot mode.
Which of the above statements are correct?
a) 1 only
b) 2 only
c) Both 1 and 2
d) Neither 1 nor 2
Q.128) Solution (d)
Called car-to-car or vehicle-to-vehicle communication, the technology lets cars broadcast
their position, speed, steering-wheel position, brake status, and other data to other vehicles
within a few hundred meters. The other cars can use such information to build a detailed

www.iasbaba.com

71

General Science and Technology- Subject wise compilation of 60 Day Plan- 2016
picture of whats unfolding around them, revealing trouble that even the most careful and
alert driver, or the best sensor system, would miss or fail to anticipate.
Already many cars have instruments that use radar or ultrasound to detect obstacles or
vehicles. But the range of these sensors is limited to a few car lengths, and they cannot see
past the nearest obstruction.
Car-to-car communication should also have a bigger impact than the advanced vehicle
automation technologies that have been more widely heralded. Though self-driving cars
could eventually improve safety, they remain imperfect and unproven, with sensors and
software too easily bamboozled by poor weather, unexpected obstacles or circumstances,
or complex city driving. Simply networking cars together wirelessly is likely to have a far
bigger and more immediate effect on road safety.
Q.129) Consider the following statements regarding Nuclear Suppliers Group:
1. It is a group of countries with nuclear weapons that seek to prevent nuclear
proliferation.
2. NSG controls the export of materials, equipment and technology that can be used to
manufacture Nuclear Weapons.
3. Recently India has become the member of NSG.
Which of the above statements are incorrect?
a) 3 only
b) 1 and 2
c) 1 and 3
d) None of the above
Q.129) Solution (c)
Nuclear Suppliers Group (NSG) is a group of nuclear supplier countries that seek to prevent
nuclear proliferation by controlling the export of materials, equipment and technology that
can be used to manufacture nuclear weapons.
All members of NSG do not have nuclear weapons.
Q.130) In International Space Station, a ball pen doesnt work becausea) There is no air pressure.
b) There is no gravity.
c) The above statement is incorrect as ball point pen does work.
d) Because of artificial pressure, the ball gets jammed.
Q.130) Solution (b)
The pen doesnt work because there is no gravity to pull the ink down.
Q.131) Consider the following

www.iasbaba.com

72

General Science and Technology- Subject wise compilation of 60 Day Plan- 2016
1. Light and Sound Waves are Electromagnetic Waves
2. Both shows the property of Diffraction, Reflection and Refraction
3. While Sound is Transverse waves, light is Longitudinal in nature
Select the correct code
a) 1, 2and 3
b) 1 and 2
c) Only 2
d) Only 3
Q.131) Solution (c)
Sound is Mechanical Wave not electromagnetic. Sound is Longitudinal and light is
Transverse. Second statement is true
Q.132) Consider the following statements:
1. The electric current in conductors is due to the flow of free protons.
2. The direction of electric current is taken as opposite to the direction of flow of the
electrons.
Which of the above statements is correct?
a) 1 only
b) 2 only
c) Both 1 and 2
d) Neither 1 nor 2
Q.132) Solution (b)
The electric current in conductors is due to the flow of free electrons. The direction of flow
of current is opposite to the direction of flow of electrons.
The electric current in conductors is due to the flow of free electrons. The direction of flow
of current is opposite to the direction of flow of electrons.
Q.133 During Ice skating, the skater increases his rotation speed by pulling in his arms. The
speed increases because ofa) Conservation of angular momentum
b) Decrease in resistance from the air
c) Shifting of the centre of mass
d) Shifting of the centre of gravity
Q.133) Solution (a)

www.iasbaba.com

73

General Science and Technology- Subject wise compilation of 60 Day Plan- 2016
The ballerina by pulling in his arms decreases the radius of rotation. This in turn reduces the
moment of inertia. To conserve angular momentum, if the radius is decreased the angular
velocity increases.
Q.134) The Cabinet Committee on Economic Affairs approved the launch of the National
Supercomputing Mission. This is a visionary program to enable India to leapfrog to the
league of world class computing power nations. Which of the following statements are
correct about National Supercomputing Mission?
1. The Mission would be implemented and steered jointly by the Department of Science
and Technology (DST) and Department of Electronics and Information Technology
(DeitY).
2. The mission aims at developing the fastest supercomputer of the world in India.
3. The Mission envisages empowering our national academic and R&D institutions spread
over the country by installing a vast supercomputing grid.
4. The Mission also includes development of highly professional High Performance
Computing (HPC) aware human resource for meeting challenges of development of
these applications.
Select the code from below:
a) 2 only
b) 1, 3 and 4
c) 1,2 and 4
d) All of the above
Q.134) Solution (b)
National Supercomputing Mission
The Cabinet Committee on Economic Affairs approved the launch of the National
Supercomputing Mission on 25th March 2015. This is a visionary program to enable India to
leapfrog to the league of world class computing power nations. The Mission would be
implemented and steered jointly by the Department of Science and Technology (DST) and
Department of Electronics and Information Technology (DeitY) at an estimated cost of
Rs.4500 crore over a period of seven years.
The Mission envisages empowering our national academic and R&D institutions spread over
the country by installing a vast supercomputing grid comprising of more than 70 highperformance computing facilities. These supercomputers will also be networked on the
National Supercomputing grid over the National Knowledge Network (NKN). The NKN is
another programme of the government which connects academic institutions and R&D labs
over a high speed network. Academic and R&D institutions as well as key user
departments/ministries would participate by using these facilities and develop applications
of national relevance. The Mission also includes development of highly professional High

www.iasbaba.com

74

General Science and Technology- Subject wise compilation of 60 Day Plan- 2016
Performance Computing (HPC) aware human resource for meeting challenges of
development of these applications.
Q.135) Consider the following statements:
1. Tuberculosis is caused by virus which is spread from person to person through air.
2. Bacille Calmette-Guerin (BCG) vaccinations are used in many countries to prevent severe
forms of TB in children.
3. Multidrug-resistant TB (MDR TB) is caused by an organism that is resistant to at least
isoniazid and rifampin, the two most potent TB drugs.
Which of the above statements are correct?
a) 1 and 2
b) 2 and 3
c) 1 and 3
d) None of the above
Q.135) Solution (b)
TB is a bacterial infection.
Q.136) Which of the following is not the property of base?
a) They react with acids to give salt and water
b) They are soapy to touch
c) They are donors of protons
d) They are acceptors of protons
Q.136) Solution (c)
Acids are substances that can donate H+ ions to bases. Since a hydrogen atom is a
proton and one electron, technically an H+ ion is just a proton. So an acid is a "proton
donor", and a base is a "proton acceptor". The reaction between an acid and base is
essentially a proton transfer.
The properties of acids and bases are:
Acids : Sour, corrosive to metals and skin, PH of <7 and a POH of >7, turns blue litmus
paper red, contains hydrogen (H+ ions), reacts with bases to form water and a salt.
Bases: Bitter, slippery or soapy, PH of >7 and a POH of <7, turns red litmus paper blue,
turns phenolphthalein pink/purple, contains hydroxide (OH- ions), reacts with acids to
form water and a salt.
Q.137) Antibodies are formed by
a) T-cells
b) Monocytes

www.iasbaba.com

75

General Science and Technology- Subject wise compilation of 60 Day Plan- 2016
c) Phagocytes
d) B-cells
Q.137) Solution (d)
Antibody, also called immunoglobulin, a protective protein produced by the immune
system in response to the presence of a foreign substance, called an antigen. Antibodies
recognize and latch onto antigens in order to remove them from the body. A wide range
of substances are regarded by the body as antigens, including disease-causing organisms
and toxic materials such as insect venom.
When an alien substance enters the body, the immune system is able to recognize it as
foreign because molecules on the surface of the antigen differ from those found in the
body. To eliminate the invader, the immune system calls on a number of mechanisms,
including one of the most importantantibody production.
Antibodies are produced by specialized white blood cells called B lymphocytes (or B
cells). When an antigen binds to the B-cell surface, it stimulates the B cell to divide and
mature into a group of identical cells called a clone. The mature B cells, called plasma
cells, secrete millions of antibodies into the bloodstream and lymphatic system.
Q.138) Green plants in the sea are useful for the respiration of fish because
a) They give out oxygen
b) They give out carbon dioxide
c) The give out oxygen and carbon dioxide simultaneously
d) They take oxygen and give out carbon dioxide
Q.138) Solution (a)
Simple direct question - Factual answer
Q.139) Which of the following have highest elasticity?
a) Steel
b) Copper
c) Rubber
d) Aluminium
Q.139) Solution (a)
The definition of elastic in physics is unfortunately inverse of common sense elastic. The
more difficult it is to stretch, the more elastic a material is called to be because elasticity
is defined by the ratio stress to strain and not vice versa.
Elasticity is measured as ratio of stress to strain also known as Youngs Modulus. For a
given stress (stretching force per unit area) strain is much smaller in steel than in rubber
and hence the answer.

www.iasbaba.com

76

General Science and Technology- Subject wise compilation of 60 Day Plan- 2016

If the same force is applied to the wire of steel and rubber thread, which are of equal
length and cross-section area, we will find that the extension in the rubber thread is
much greater than extension in steel wire. Therefore, for a given stress, the strain
produced in steel is much smaller than that produced in the rubber. This implies that
Youngs modulus for steel is greater than that for rubber. Therefore, steel is more elastic
than rubber.
And also physics defines elasticity as "resistance to change". The greater the resistance
to change, the greater is the elasticity of the material and the faster it comes back to its
original shape or configuration when the deforming force is removed. By this definition,
steel is more elastic than rubber because steel comes back to its original shape faster
than rubber when the deforming forces are removed.

Q.140) Consider the following statements:


1) The Global Apollo Programme is a call for a major global science and economics research
programme to make carbon-free baseload electricity less costly than electricity from
coal by the year 2025.
2) Key areas of focus includes Renewable energy, Energy storage, Smart grids and
hydrogen vehicles.
Which of the above given statement(s) is/are true?
a) 1 only
b) 2 only
c) Both
d) None
Q.140) Solution (c)
Both the given statements are correct and self-explanatory.
Key areas of focus:
Renewable energy - in particular that derived from solar and wind sources
Energy storage
Smart grids
hydrogen vehicles

Q.141) Hydra, a multi-cellular invertebrate of phylum cnidarian (coelenterata) can give


rise to new offspring by various methods. Choose the method by which the offspring are
produced with significant variations.
a) budding
b) regeneration

www.iasbaba.com

77

General Science and Technology- Subject wise compilation of 60 Day Plan- 2016
c) sexual reproduction
d) asexual reproduction
Q.141) Solution (c)
*Note: Answer to this question was given wrong and is now corrected.
Explanation:
Hydras are a type of freshwater cnidarian related to jellyfish, sea anemones and corals.
Hydras reproduce asexually by developing small, genetically identical polyps that
protrude from the parent. These polyps break off from the parent to form a new
organism in a process called "budding."
i.e. Hydras generally reproduce asexually. They do this through budding.
If the environmental conditions get bad enough, they are likely to switch to sexual
reproduction, which increases the genetic variation in the population.
Since, the question asks for the method by which the offspring are produced with
significant variations the correct answer is through sexual reproduction (option c)
Asexual reproduction produces very closely resembling offspring. Asexual reproduction
thus results in offspring with minor variations. Sexually reproducing organisms produce
offspring with marked, significant and visible variations.

Q.142) Live Vaccines in current use include:


1. BCG which is an attenuated Bovine strain of the tubercle bacillus
2. Oral Polio virus vaccine
3. TAB vaccine
4. Smallpox vaccine prepared from cowpox virus.
Which of the above statements are correct?
a) 1 and 2
b) 1,2 and 3
c) 1,3 and 4
d) All of the above
Q.142) Solution (a)
TAB vaccine suspension of killed typhoid and paratyphoid A and B bacilli.
Smallpox has been eradicated.
Q.143) Among the following organisms list out the asexually reproducing organisms:
1) Paramoecium

www.iasbaba.com

78

General Science and Technology- Subject wise compilation of 60 Day Plan- 2016
2) Euglena
3) Earthworm
4) Bird
Choose the appropriate code:
a) 1 and 3 only
b) 1 and 2 only
c) 2 and 3 only
d) 2, 3 and 4 only
Q.143) Solution (b)
Explanation:
Paramoecium and EugIena are the asexually reproducing organisms.
Q.144) Which of the following is a hereditary disease?
a) Kwashiorkor
b) Ringworm
c) Down's syndrome
d) Polio
Q.144) Solution (c)
Explanation:

www.iasbaba.com

79

General Science and Technology- Subject wise compilation of 60 Day Plan- 2016
Q.145) Which among the following regulates and controls the amount of light entering the
eye?
a) pupil
b) retina
c) lens
d) iris
Q.145) Solution (d)
Explanation:
The iris is the ring of pigmented tissue surrounding the pupil that varies in color. The iris
opens and closes to control the amount of light entering the eye through the pupil.
The pupil is the opening in the center of the iris where light enters the eye. When
looking at the eye, the pupil appears black. In dim light, the opening in the center of the
iris expands to allow more light to enter the pupil. In bright light, this opening constricts
to decrease the amount of light entering the eye to protect it from damage.

Q.146) Consider the following physical quantities


1. Gravitational Potential
2. Impulse
3. Power
4. Change in temperature

www.iasbaba.com

80

General Science and Technology- Subject wise compilation of 60 Day Plan- 2016

Which of the following are vector quantities?


a) 1 and 2
b) Only 2
c) 2 and 4
d) 1,2 and 3
Q.146) Solution (c)
Scalar is the measurement of a medium strictly in magnitude.
Vector is a measurement that refers to both the magnitude of the medium as well as the
direction of the movement the medium has taken.
Impulse is a term that quantifies the overall effect of force acting over time. Since force is a
vector quantity, impulse is also a vector in the same direction.
Temperature is a scalar quantity. But change in anything is a vector quantity because it can
go in any direction. The measurement of the increase or decrease in the mediums
temperature is a vector quantity.
Gravitational Potential and Power can be fully described by the magnitude or a numerical
value alone.
Q.147) Project ReAnima is concerned with which of the following
a) Revival of brain dead patients
b) Macrocytic Anaemia
c) A project of Disney India and Toonz Animation
d) Retina Display
Q.147) Solution (a)
It is the worlds first clinical trial on the revival of brain dead patients
It is a Joint Venture between US company and a Rudrapur (Uttrakhand) based hospital
India currently has no laws for clinical trials on living cadaver or brain dead patients
And since India has no laws, no permissions are required
It will study the effects of stem cell therapy, Median Nerve Stimulation and a laser
therapy on comatose patients
http://www.thehindu.com/news/national/a-strange-project-to-revive-the-braindead/article8601787.ece
Q.148) Consider following statements with respect to CARTOSAT-2
1. It is an Earth Observation Satellite
2. Its orbit is Polar Sun Synchronous
3. It was launched by GSLV
Which of the statements are correct?

www.iasbaba.com

81

General Science and Technology- Subject wise compilation of 60 Day Plan- 2016
a)
b)
c)
d)

Only 1
1 and 2
1 and 3
All of the above

Q.148) Solution (b)


http://www.thehindu.com/sci-tech/science/isro-all-set-for-wednesdays-20in1mission/article8756559.ece
http://www.isro.gov.in/Spacecraft/cartosat-2-0
Q.149) Consider the following statements with respect to Lithium-ion battery
1. The anode is generally made of carbon and cathode of metal oxide
2. Lithium ions move from cathode to anode during discharge
Select the incorrect statement/s
a) Only 1
b) Only 2
c) Both
d) None
Q.149) Solution (a)
Generally, the negative electrode (cathode) of a conventional lithium-ion cell is made from
carbon. The positive electrode (anode) is a metal oxide, and the electrolyte is a lithium salt
in an organic solvent.
A lithium-ion battery (sometimes Li-ion battery or LIB) is a member of a family of
rechargeable battery types in which lithium ions move from the negative electrode
(cathode) to the positive electrode (anode) during discharge and back when charging. Liion batteries use an intercalated lithium compound as one electrode material, compared to
the metallic lithium used in a non-rechargeable lithium battery. The electrolyte, which
allows for ionic movement, and the two electrodes are the constituent components of a
lithium-ion battery cell.
Q.150) The ripening of fruits can be accelerated by:
a) Decreasing oxygen concentration of the surrounding
b) Increasing ethylene concentration of the surrounding
c) Reducing water supply to the plants
d) Spraying urea during maturation of fruits
Q.150) Solution (b)
Ethylene helps in the ripening of the fruit.

www.iasbaba.com

82

General Science and Technology- Subject wise compilation of 60 Day Plan- 2016
Q.151) The chief poisonous snakes of India are:
a) Cobra, Krait, Python, Wolf-snake
b) Sea Snake, Pit Viper, Krait, Cobra, Rat-snake
c) Pit Viper, Russels Viper, Cobra, Krait, Sea snake
d) Python, pit viper, cobra, Krait, Russels viper
Q.151) Solution (c)
Python and Rat-snake are not poisonous.
Q.152) An antibiotic is:
a) A chemical compound produced by a living organism that inhibits the growth of the
other organisms.
b) A compound synthesized by a living organisms that inhibits the growth of microbes.
c) A synthetic compound inhibiting the growth of other organisms
d) A synthetic compound inhibiting the growth of bacteria
Q.152) Solution (b)
Antibiotics inhibit the growth of microbes not organisms.
Q.153) Which of the following is used as a lubricant in heavy machines?
a) Bauxite
b) Sulphur
c) Phosphorus
d) Graphite
Q.153) Solution (d)
Graphite has a layered structure where layers can easily slide over each other reducing
friction between the moving parts of the machine.
Q.154) A hydrogen filled balloon,
a) Always rises in air
b) Rises only if its total weight is much less than the weight of air it displaces
c) Rises only if its total weight is equal to the weight of air it displaces.
d) Can never rise in the air
Q.154) Solution (b)
A hydrogen filled balloon works on the principle of Archimedes principle. It will rise as long
as the total weight of air displaced by balloon is more than the total weight of balloon.
Q.155) The earth and the moon are about the same distance from the sun, yet on the
average the earth is much warmer than the moon. Why?

www.iasbaba.com

83

General Science and Technology- Subject wise compilation of 60 Day Plan- 2016
a)
b)
c)
d)

The moon is much smaller than the earth


Nights on the moon are much longer
The moon has almost no atmosphere
The surface of the moon is darker

Q.155) Solution (c)


Since there is no atmosphere, there is no green house effect to trap the heat.
Q.156) In which of the following cases, a person will feel weightlessness?
1. He is falling freely
2. He is orbiting in a satellite
3. He is in an aeroplane flying at a high altitude
4. He is having ride in a gas filled balloon
Select the code from the following:
a) 2 only
b) 1 and 2
c) 3 and 4
d) 1,2 and 3
Q.156) Solution (b)
A person feels weightless during a free fall and in space.
Note: Initially astronauts were trained for weightlessness in free falling lifts.
Q.157) Stars appear to move from East to West because
a) The universe is moving from east to west.
b) The earth is revolving round the sun
c) The earth is rotating from East to West
d) The Earth is rotating from West to East
Q.157) Solution (d)
Since earth rotates from west to east, relatively stars appear to move from east to west.
Q.158) Compared to the velocity of light, the velocity of radiowaves is:
a) Smaller
b) Greater
c) The same
d) Depends on the type of radio waves
Q.158) Solution (c)

www.iasbaba.com

84

General Science and Technology- Subject wise compilation of 60 Day Plan- 2016
Both the light waves and radio waves are electromagnetic radiations and move with same
velocity.
Q.159) In an electric bulb, a little Nitrogen or Argon is introduced at a low pressure to
a) Cool the bulbs filament
b) Prevent the evaporation of the filament
c) Withstand atmospheric pressure
d) Increase the brightness of the filament
Q.159) Solution (b)
Nitrogen or Argon is added in the bulb to create an inert environment so that the filament
does not react. At this high temperature filament can react with oxygen and evaporate.
Q.160) Consider the following about Viruses
1. They evolve by metabolism
2. They do not possess genes
3. They cannot replicate
Select the correct code
a) 1 and 2
b) Only 2
c) 1, 2 and 3
d) None
Q.160) Solution (d)
Viruses have genes. They do not metabolize but replicate by creating multiple copies of
themselves through self-assembly.
Q.161) Consider the following about Phosphorous
1. White Phosphorous is crystalline
2. Red Phosphorous is polymeric
3. Black Phosphorous is a semiconductor
Select the correct code
a) 1 and 2
b) Only 3
c) 1 and 3
d) 1, 2 and 3
Q.161) Solution (d)
All the statements are correct

www.iasbaba.com

85

General Science and Technology- Subject wise compilation of 60 Day Plan- 2016

Q.162) Consider the following about Geostationary Orbits


1. There are various geostationary orbits
2. The orbits inclination with earth is not zero
3. The orbit is in equatorial plane
Select the incorrect code
a) 1 and 2
b) Only 1
c) 2 and 3
d) Only 1
Q.162) Solution (a)
There is only one geostationary orbit. Since it is in equatorial plane, the inclination is zero.
Q.163) Cholesterols provide strengths and flexibility to cell membranes. Consider the
following w.r.t Cholesterols
1. They are highly soluble in water
2. They are subclass of lipids known as steroids
3. It is an important precursor molecule for the synthesis of Vitamin D
Select the correct code
a) 1 and 2
b) 2 and 3
c) 1, 2 and 3
d) 1 and 3
Q.163) Solution (b)
They are slightly soluble in water. 2nd and 3rd statements are correct
Q.164) Consider the following
1. Antifreeze
2. Antiseptics
3. Preservatives
4. Fuels
Which of the above can be the application of an Alcohol?
a) 1, 3 and 4
b) 2, 3 and 4
c) 1, 2 and 4
d) 1, 2, 3 and 4

www.iasbaba.com

86

General Science and Technology- Subject wise compilation of 60 Day Plan- 2016

Q.164) Solution (d)


50% v/v solution of Ethylene Glycol in water is commonly used as antifreeze
Ethanol is used as an antiseptic.
Q.165) Exomars is the Mars Mission launched jointly by
a) Russia and China
b) USA and Russia
c) Europe and Russia
d) Europe and USA
Q.165) Solution (c)
Direct Question.
Q.166) Consider the following about Goldilocks Zone
a) Habitable zone in the planetary system where temperature is neither too high nor too
low
b) Habitable zone in the planetary system where temperature is very high
c) Habitable zone in the planetary system where temperature is very low
d) None
Q.166) Solution (a)
Q.157) Coke is one of the materials of the charge added to blast furnace for the
production of steel/iron. Its function is to
1. Act as a reducing agent.
2. Function as fuel, to supply heat.
3. Act as an oxidizing agent.
Of these statements
a) 1 and 2
b) 2 and 3
c) None
d) All are correct
Q.157) Solution (a)
Coke acts as a reducing agent not an oxidising agent.
Q.158) Consider the following statements
1. Pinaka is Multi Barrel Rocket Launcher developed by DRDO
2. INSAS is a small satellite developed by ISRO

www.iasbaba.com

87

General Science and Technology- Subject wise compilation of 60 Day Plan- 2016
3. Nag is an antitank missile
Select the correct statement/s
a) 1 and 3
b) 2 and 3
c) 1 and 2
d) All
Q.158) Solution (a)
The INSAS is a family of infantry arms consisting of an assault rifle and a light machine gun
(LMG)
Q.159) Consider the following
1. A person having blood group, B+ can donate blood to B- and O+
2. A person having blood group AB+ can receive blood from everyone
3. A person having blood group O- can donate blood to everyone
Select the correct answer using codes
a) 1, 2 and 3
b) 1 and 3
c) 2 and 3
d) Only 3
Q.159) Solution (c)
Q.160) Consider the following statements with respect to Transformative Carbon Asset
Facility (TCAP)
1. It is launched by United Nations Environment Programme
2. It will help developing countries pay for emission reductions and combat climate change
3. Germany, Norway, Sweden, and Switzerland are part of TCAP
Which of the following statements is/are correct?
a) 1 and 2
b) Only 2
c) 2 and 3
d) All of the above
Q.160) Solution (c)
The World Bank has launched a $500 million dollars Transformative Carbon Asset Facility
(TCAF) to help developing countries pay for emission reductions and combat climate
change.

www.iasbaba.com

88

General Science and Technology- Subject wise compilation of 60 Day Plan- 2016
Announced by Germany, Norway, Sweden, and Switzerland - have jointly pledged an initial
250 million dollars investment.
http://www.worldbank.org/en/news/press-release/2015/11/30/new-500-millioninitiative-to-boost-large-scale-climate-action-in-developing-countries
Q.161) Consider following statements with respect to LCA Tejas
1. It is jointly developed by Aeronautical Development Agency and DRDO
2. It is the smallest lightweight, multi-role, single-engine tactical fighter aircraft in the
world
Select the correct statement(s)
a) Only 1
b) Only 2
c) Both 1 and 2
d) None
Q.161) Solution (b)
It is jointly developed by Aeronautical Development Agency and Hindustan Aeronautics
Limited
It is the smallest lightweight, multi-role, single-engine tactical fighter aircraft in the world
http://www.tejas.gov.in/specifications.html
Q.162) Which of the following is referred to as Wasting?
a) Low weight-for-height
b) Low height-for-age
c) Low weight-for-age
d) High weight-for-height
Q.162) Solution (a)
Low weight-for-height: Wasting or thinness indicates in most cases a recent and severe
process of weight loss, which is often associated with acute starvation and/or severe
disease.
Low height-for-age: Stunted growth reflects a process of failure to reach linear growth
potential as a result of suboptimal health and/or nutritional conditions.
Low weight-for-age: Weight-for-age reflects body mass relative to chronological age. It is
influenced by both the height of the child (height-for-age) and his or her weight (weight-forheight), and its composite nature makes interpretation complex.
High weight-for-height: "Overweight" is the preferred term for describing high weight-forheight.
http://www.who.int/nutgrowthdb/about/introduction/en/index2.html

www.iasbaba.com

89

General Science and Technology- Subject wise compilation of 60 Day Plan- 2016
Q.163) Consider the following statements with respect to Google Street View?
1. It provides panoramic views from positions along many streets across the world
2. It uses image stitching technology
3. India denied permission for the service citing security issues
Select the correct statement(s)
a) Only 1
b) 1 and 3
c) 1 and 2
d) All of the above
Q.163) Solution (d)
Google Street View is a technology featured in Google Maps and Google Earth that provides
panoramic views from positions along many streets in the world.
Google Street View displays panoramas of stitched images
Image stitching or photo stitching is the process of combining multiple photographic images
with overlapping fields of view to produce a segmented panorama or high-resolution image.
Q.164) Consider the following statements concerning Astrobiology Mission
1. India is part of the Spaceward Bound programme for the first time
2. Ladakh has been selected as the first site because it offers high UV (ultra-violet)
exposed, dry ecosystem with Mars analogue topological features
Select the incorrect statement(s)
a) Only 1
b) Only 2
c) Both 1 and 2
d) None of the above
Q.164) Solution (d)
Both statements are correct.
NASA and ISRO will mount an expedition to Ladakh to study the similarities of certain parts
of the regions topography and microbial life to Martian surroundings.
For the first time India is part of Spaceward Bound programme, which funds expeditions to
places with extreme climate conditions.
Ladakh offers a high UV exposed, dry ecosystem with Mars analogue topological features.
It tell us a lot about the origin and evolution of our planets topological features
http://spacewardbound.astrobiologyindia.in/home/spaceward-bound-india/

www.iasbaba.com

90

General Science and Technology- Subject wise compilation of 60 Day Plan- 2016
Q.165) India will be getting its first High Speed Train from a Spanish Company called
Taglo. The company is globally known for faster and lighter trains. Consider the following
statements:
1. The train will be tested on Delhi-Mumbai Railway line.
2. It can be run at a speed of around 300 km/hr without any significant upgrade of railway
tracks.
3. Currently the fastest train of India is Gatiman Express which runs between Delhi and Agra.
Which of the above statements are correct?
a) 1 and 2
b) 2 and 3
c) 1 and 3
d) All of the above
Q.165) Solution (c)
The company has offered lighter and safer trains, which can be run at speeds of about 160
km an hour and without any major upgrade of existing tracks. The fastest train on Indian
Railways system, Gatimaan Express, runs at 160 km an hour between Delhi and Agra.
*Note: Question has been corrected for factual error. Taglo is a high speed train not a
bullet train.

www.iasbaba.com

91

S-ar putea să vă placă și